HistoPath Flashcards

Questions from textbook and blackboard

1
Q
A 36-year-old man presents to accident and emergency with a 1-day history of a fever of 39.2 C and night sweats. A new heart murmur is detected by the on- call cardiologist. The patient admits to being an intravenous drug user.
A Monckeberg arteriosclerosis
B Infective endocarditis
C Dressler’s syndrome
D Dilated cardiomyopathy
E Rheumatic heart disease
F Left heart failure
G Hypertrophic obstructive cardiomyopathy
H Aortic stenosis
I Carcinoid syndrome
A

B Infective endocarditis

How well did you know this?
1
Not at all
2
3
4
5
Perfectly
2
Q
A 64-year-old man presents to accident and emergency due to a collapse at home. An ejection systolic murmur is heard at the upper-left sternal edge.
A Monckeberg arteriosclerosis
B Infective endocarditis
C Dressler’s syndrome
D Dilated cardiomyopathy
E Rheumatic heart disease
F Left heart failure
G Hypertrophic obstructive cardiomyopathy
H Aortic stenosis
I Carcinoid syndrome
A

H Aortic stenosis

How well did you know this?
1
Not at all
2
3
4
5
Perfectly
3
Q
A widowed 72-year-old woman who has passed away at home is sent for autopsy due to unknown cause of death. Post-mortem examination reveals a nutmeg liver and haemosiderin-laden macrophages in the lungs.
A Monckeberg arteriosclerosis
B Infective endocarditis
C Dressler’s syndrome
D Dilated cardiomyopathy
E Rheumatic heart disease
F Left heart failure
G Hypertrophic obstructive cardiomyopathy
H Aortic stenosis
I Carcinoid syndrome
A

F Left heart failure

Pulmonary congestion with dilated capillaries and leakage of blood into alveolar spaces leads to an increase in hemosiderin-laden macrophages

How well did you know this?
1
Not at all
2
3
4
5
Perfectly
4
Q
A 54-year-old man presents to accident and emergency with fever and pleuritic chest pain. It is noted that the patient suffered a myocardial infarction 4 weeks previously
A Monckeberg arteriosclerosis
B Infective endocarditis
C Dressler’s syndrome
D Dilated cardiomyopathy
E Rheumatic heart disease
F Left heart failure
G Hypertrophic obstructive cardiomyopathy
H Aortic stenosis
I Carcinoid syndrome
A

C Dressler’s syndrome

How well did you know this?
1
Not at all
2
3
4
5
Perfectly
5
Q
A 46-year-old man is referred to the cardiology outpatient clinic. On investiga- tion he is found to have mitral regurgitation and has a past history of St Vitus Dance when he was in school and a mild pericarditis.
A Monckeberg arteriosclerosis
B Infective endocarditis
C Dressler’s syndrome
D Dilated cardiomyopathy
E Rheumatic heart disease
F Left heart failure
G Hypertrophic obstructive cardiomyopathy
H Aortic stenosis
I Carcinoid syndrome
A

E Rheumatic heart disease

How well did you know this?
1
Not at all
2
3
4
5
Perfectly
6
Q
A 40-year-old male presents to his GP with chronic cough with copious amounts of purulent mucus production. High resolution CT scans demonstrate dilated bronchi.
A Hyaline membrane disease
B Small cell carcinoma
C Extrinsic allergic alveolitis
D Bronchiectasis
E Non-small cell carcinoma
F Chronic bronchitis 
G Pulmonary oedema 
H Cystic fibrosis
I Sarcoidosis
A

D Bronchiectasis

How well did you know this?
1
Not at all
2
3
4
5
Perfectly
7
Q
A 14-year-old girl is admitted to hospital after suffering her third bout of pneumonia caused by Pseudomonas aeruginosa infection. She also has a previous admission for pancreatitis.
A Hyaline membrane disease
B Small cell carcinoma
C Extrinsic allergic alveolitis
D Bronchiectasis
E Non-small cell carcinoma
F Chronic bronchitis 
G Pulmonary oedema 
H Cystic fibrosis
I Sarcoidosis
A

H Cystic fibrosis

How well did you know this?
1
Not at all
2
3
4
5
Perfectly
8
Q
A 58-year-old man presents to his GP with haemoptysis and weight loss. He has a 30 pack–year history of smoking. He is referred to the oncologist for a biopsy, who determines ‘oat-shaped’ cells on microscopy.
A Hyaline membrane disease
B Small cell carcinoma
C Extrinsic allergic alveolitis
D Bronchiectasis
E Non-small cell carcinoma
F Chronic bronchitis 
G Pulmonary oedema 
H Cystic fibrosis
I Sarcoidosis
A

B Small cell carcinoma

How well did you know this?
1
Not at all
2
3
4
5
Perfectly
9
Q
A 62-year-old man presents to his GP with shortness of breath, lethargy and weight loss. The patient’s chest X-ray reveals a peripheral focal lesion in the left lung field.
A Hyaline membrane disease
B Small cell carcinoma
C Extrinsic allergic alveolitis
D Bronchiectasis
E Non-small cell carcinoma
F Chronic bronchitis 
G Pulmonary oedema 
H Cystic fibrosis
I Sarcoidosis
A

E Non-small cell carcinoma

How well did you know this?
1
Not at all
2
3
4
5
Perfectly
10
Q
A 53-year-old woman with a history of rheumatic fever presents to accident and emergency with severe shortness of breath, and has been coughing up pink frothy sputum for the past 2 days.
A Hyaline membrane disease
B Small cell carcinoma
C Extrinsic allergic alveolitis
D Bronchiectasis
E Non-small cell carcinoma
F Chronic bronchitis 
G Pulmonary oedema 
H Cystic fibrosis
I Sarcoidosis
A

G Pulmonary oedema

How well did you know this?
1
Not at all
2
3
4
5
Perfectly
11
Q
A 35-year-old woman is referred to the rheumatology clinic due to recent onset dysphagia. The patient also reports that her fingers have turned very pale and cold. One examination she is found to have tightening of the skin near her fin- ger tips and small dilated vessels on her skin.
A Systemic lupus erythematosus
B Sjögren’s syndrome
C Diffuse scleroderma
D Amyloidosis
E Takayasu arteritis
F Dermatomyositis
G CREST syndrome
H Polymyositis
I Microscopic polyangitis
A

G CREST syndrome

How well did you know this?
1
Not at all
2
3
4
5
Perfectly
12
Q
A 35-year-old woman with a history of recurrent miscarriages presents to her GP with joint pains. Blood tests reveal she is anti-double stranded DNA antibody positive.
A Systemic lupus erythematosus
B Sjögren’s syndrome
C Diffuse scleroderma
D Amyloidosis
E Takayasu arteritis
F Dermatomyositis
G CREST syndrome
H Polymyositis
I Microscopic polyangitis
A

A Systemic lupus

How well did you know this?
1
Not at all
2
3
4
5
Perfectly
13
Q
A 68-year-old man presents to accident and emergency with symptoms sugges- tive of heart failure. All initial investigations do not determine an underlying cause. However, a tongue biopsy sample gains an apple-green birefringence under polarized light using Congo red stain.
A Systemic lupus erythematosus
B Sjögren’s syndrome
C Diffuse scleroderma
D Amyloidosis
E Takayasu arteritis
F Dermatomyositis
G CREST syndrome
H Polymyositis
I Microscopic polyangitis
A

D Amyloidosis

How well did you know this?
1
Not at all
2
3
4
5
Perfectly
14
Q
A 45-year-old woman presents to accident and emergency with signs suggestive of renal failure. She is found to be p-ANCA positive.
A Systemic lupus erythematosus
B Sjögren’s syndrome
C Diffuse scleroderma
D Amyloidosis
E Takayasu arteritis
F Dermatomyositis
G CREST syndrome
H Polymyositis
I Microscopic polyangitis
A

I Microscopic polyangitis

How well did you know this?
1
Not at all
2
3
4
5
Perfectly
15
Q
A 52-year-old man presents to his GP with limb weakness and shortness of breath. A distinctive rash is noted around both eyes as well as plaques on the joints of his hands.
A Systemic lupus erythematosus
B Sjögren’s syndrome
C Diffuse scleroderma
D Amyloidosis
E Takayasu arteritis
F Dermatomyositis
G CREST syndrome
H Polymyositis
I Microscopic polyangitis
A

F Dermatomyositis

How well did you know this?
1
Not at all
2
3
4
5
Perfectly
16
Q
A 54-year-old man is seen in the neurology clinic due to tremor and rigidity. A DAT scan reveals reduced uptake in the substantia nigra.
A Subarachnoid haemorrhage
B Parkinson’s disease
C Extradural haemorrhage
D Vascular dementia
E Subdural haemorrhage
F Intracerebral haemorrhage G Multiple sclerosis
H Duret haemorrhage
I Alzheimer’s disease
A

B Parkinson’s disease

How well did you know this?
1
Not at all
2
3
4
5
Perfectly
17
Q
A 74-year-old man presents to accident and emergency with increasing head- ache and confusion. The man’s wife suggests her husband may have tripped and fallen 3 days previously.
A Subarachnoid haemorrhage
B Parkinson’s disease
C Extradural haemorrhage
D Vascular dementia
E Subdural haemorrhage
F Intracerebral haemorrhage G Multiple sclerosis
H Duret haemorrhage
I Alzheimer’s disease
A

E Subdural haemorrhage

How well did you know this?
1
Not at all
2
3
4
5
Perfectly
18
Q
A 45-year-old woman presents to accident and emergency with the worst head- ache she has ever experienced. She is noted to have polycystic kidney disease.
A Subarachnoid haemorrhage
B Parkinson’s disease
C Extradural haemorrhage
D Vascular dementia
E Subdural haemorrhage
F Intracerebral haemorrhage G Multiple sclerosis
H Duret haemorrhage
I Alzheimer’s disease
A

A Subarachnoid haemorrhage

How well did you know this?
1
Not at all
2
3
4
5
Perfectly
19
Q
A 35-year-old woman presents to the neurology clinic with weakness of her left side. On examination she is found to have nystagmus and an intention tremor. The patient complains of blurred vision for the past month.
A Subarachnoid haemorrhage
B Parkinson’s disease
C Extradural haemorrhage
D Vascular dementia
E Subdural haemorrhage
F Intracerebral haemorrhage G Multiple sclerosis
H Duret haemorrhage
I Alzheimer’s disease
A

G Multiple sclerosis

How well did you know this?
1
Not at all
2
3
4
5
Perfectly
20
Q
A 42-year-old man who suffers from Down syndrome is brought to see his GP by his carer. The carer describes how the patient has been wandering out of the house with increased frequency as well as becoming uncharacteristically aggres- sive, especially in the evening.
A Subarachnoid haemorrhage
B Parkinson’s disease
C Extradural haemorrhage
D Vascular dementia
E Subdural haemorrhage
F Intracerebral haemorrhage G Multiple sclerosis
H Duret haemorrhage
I Alzheimer’s disease
A

I Alzheimer’s disease

How well did you know this?
1
Not at all
2
3
4
5
Perfectly
21
Q
A 35-year-old man has a 3-week history of bloody stools without mucus with associated weight loss. A biopsy of the gastrointestinal tract reveals non- caseating granulomas with transmural inflammation.
A Ulcerative colitis
B Chronic gastritis
C Oesophageal cancer
D Coeliac disease
E Gastric carcinoma
F Barrett’s oesophagus 
G Gardener’s syndrome 
H Crohn’s disease
I Peptic ulcer disease
A

H Crohn’s disease

How well did you know this?
1
Not at all
2
3
4
5
Perfectly
22
Q
A 24-year-old woman presents to her GP with a 2-week history of diarrhoea, weight loss and fatigue. Biopsy of the gastrointestinal tract reveals villous atro- phy with crypt hyperplasia.
A Ulcerative colitis
B Chronic gastritis
C Oesophageal cancer
D Coeliac disease
E Gastric carcinoma
F Barrett’s oesophagus 
G Gardener’s syndrome 
H Crohn’s disease
I Peptic ulcer disease
A

D Coeliac disease

How well did you know this?
1
Not at all
2
3
4
5
Perfectly
23
Q
A 54-year-old man presents to his GP with a 2-week history of worsening dys- phagia. The patient’s past medical history reveals severe gastro-oesophageal reflux disease. A duodenoscopy suggests metaplastic transformation of the lower oesophageal region.
A Ulcerative colitis
B Chronic gastritis
C Oesophageal cancer
D Coeliac disease
E Gastric carcinoma
F Barrett’s oesophagus 
G Gardener’s syndrome 
H Crohn’s disease
I Peptic ulcer disease
A

F Barrett’s oesophagus

How well did you know this?
1
Not at all
2
3
4
5
Perfectly
24
Q
A 45-year-old man is referred to the gastroenterology outpatient clinic due to severe epigastric pain and an episode of haematemesis. Further testing reveals he is Helicobacter pylori positive and has a 20 pack–year history of smoking.
A Ulcerative colitis
B Chronic gastritis
C Oesophageal cancer
D Coeliac disease
E Gastric carcinoma
F Barrett’s oesophagus 
G Gardener’s syndrome 
H Crohn’s disease
I Peptic ulcer disease
A

I Peptic ulcer disease

How well did you know this?
1
Not at all
2
3
4
5
Perfectly
25
Q
A 56-year-old man presents to his GP with abdominal pain, weight loss and fatigue. A duodenoscopy allows a biopsy of a gastric lesion to be taken, which demonstrates signet ring cells and linitis plastica.
A Ulcerative colitis
B Chronic gastritis
C Oesophageal cancer
D Coeliac disease
E Gastric carcinoma
F Barrett’s oesophagus 
G Gardener’s syndrome 
H Crohn’s disease
I Peptic ulcer disease
A

E Gastric carcinoma

How well did you know this?
1
Not at all
2
3
4
5
Perfectly
26
Q
A 56-year-old man with previous history of hepatitis C infection presents to accident and emergency with jaundice. His wife notes that he has recently been bruising very easily. Ultrasound of the patient’s liver reveals irregular echo- genicity demonstrating nodules.
A Cholangiocarcinoma
B Cirrhosis
C   1-Antitrypsin deficiency
D Haemosiderosis
E Primary biliary cirrhosis
F Haemochromatosis
G Hepatocellular carcinoma
H Primary sclerosing cholangitis 
I Wilson’s disease
A

B Cirrhosis

How well did you know this?
1
Not at all
2
3
4
5
Perfectly
27
Q
A 35-year-old man presents to his GP with his mother with signs of Parkinsonism (tremor, rigidity and slow movement) as well as recent changes in his behaviour.
A Cholangiocarcinoma
B Cirrhosis
C   1-Antitrypsin deficiency
D Haemosiderosis
E Primary biliary cirrhosis
F Haemochromatosis
G Hepatocellular carcinoma
H Primary sclerosing cholangitis 
I Wilson’s disease
A

I Wilson’s disease

How well did you know this?
1
Not at all
2
3
4
5
Perfectly
28
Q
A 56-year-old woman is investigated by the hepatology team for decompensat- ed liver disease. A liver biopsy sample stains blue with Perl’s Prussian blue stain.
A Cholangiocarcinoma
B Cirrhosis
C   1-Antitrypsin deficiency
D Haemosiderosis
E Primary biliary cirrhosis
F Haemochromatosis
G Hepatocellular carcinoma
H Primary sclerosing cholangitis 
I Wilson’s disease
A

F Haemochromatosis

How well did you know this?
1
Not at all
2
3
4
5
Perfectly
29
Q
A 53-year-old man who has recently emigrated from sub-Saharan Africa is referred to the hepatology department due to recent onset weight loss, jaundice and ascites. There is history of previous aflatoxin exposure.
A Cholangiocarcinoma
B Cirrhosis
C   1-Antitrypsin deficiency
D Haemosiderosis
E Primary biliary cirrhosis
F Haemochromatosis
G Hepatocellular carcinoma
H Primary sclerosing cholangitis 
I Wilson’s disease
A

G Hepatocellular carcinoma

How well did you know this?
1
Not at all
2
3
4
5
Perfectly
30
Q
A 45-year-old woman presents to accident and emergency with jaundice and pruritis. Xanthelasma are noted on examination. The patient is found to be anti- mitochondrial antibody positive.
A Cholangiocarcinoma
B Cirrhosis
C   1-Antitrypsin deficiency
D Haemosiderosis
E Primary biliary cirrhosis
F Haemochromatosis
G Hepatocellular carcinoma
H Primary sclerosing cholangitis 
I Wilson’s disease
A

E Primary biliary cirrhosis

How well did you know this?
1
Not at all
2
3
4
5
Perfectly
31
Q
A 65-year-old man presents to his GP with blisters along his left arm that are about 1.0 cm in diameter. Gentle rubbing of the affected area does not lead to skin exfoliation.
A Pemphigoid
B Bowen’s disease
C Pityriasis rosea
D Lichen planus
E Actinic keratosis
F Psoriasis
G Basal cell carcinoma
H Erythema multiforme 
I Malignant melanoma 
J Pemphigus
A

A Pemphigoid

How well did you know this?
1
Not at all
2
3
4
5
Perfectly
32
Q
A 38-year-old man on the respiratory ward has been diagnosed with Mycoplasma pneumoniae and develops a number of target shaped rashes on his body.
A Pemphigoid
B Bowen’s disease
C Pityriasis rosea
D Lichen planus
E Actinic keratosis
F Psoriasis
G Basal cell carcinoma 
H Erythema multiforme 
I Malignant melanoma 
J Pemphigus
A

H Erythema multiforme

How well did you know this?
1
Not at all
2
3
4
5
Perfectly
33
Q
A 45-year-old woman presents to her GP with salmon-pink plaques with a silver–white scale on the extensor surfaces of her elbows.
A Pemphigoid
B Bowen’s disease
C Pityriasis rosea
D Lichen planus
E Actinic keratosis
F Psoriasis
G Basal cell carcinoma 
H Erythema multiforme 
I Malignant melanoma 
J Pemphigus
A

F Psoriasis

How well did you know this?
1
Not at all
2
3
4
5
Perfectly
34
Q
A 54-year-old man is referred to the dermatologist with a brown warty lesion on his nose which has a rough consistency. Biopsy of the lesion reveals solar elastosis.
A Pemphigoid
B Bowen’s disease
C Pityriasis rosea
D Lichen planus
E Actinic keratosis
F Psoriasis
G Basal cell carcinoma 
H Erythema multiforme 
I Malignant melanoma 
J Pemphigus
A

E Actinic keratosis

How well did you know this?
1
Not at all
2
3
4
5
Perfectly
35
Q
A 59-year-old woman presents to her dermatologist with a 3 cm black irregular lesion on her cheek. Over the next month the lesion spreads to cover 6 cm with new onset pain.
A Pemphigoid
B Bowen’s disease
C Pityriasis rosea
D Lichen planus
E Actinic keratosis
F Psoriasis
G Basal cell carcinoma 
H Erythema multiforme 
I Malignant melanoma 
J Pemphigus
A

I Malignant melanoma

How well did you know this?
1
Not at all
2
3
4
5
Perfectly
36
Q
A 45-year-old man presents to accident and emergency with haematuria and admits to passing less urine than previously. He is found to be hypertensive. Microscopy of the patient’s urine reveals red and white cell casts.
A Nephritic syndrome
B Wegener’s granulomatosis
C Membranous glomerulonephritis
D Acute tubular necrosis
E Minimal change glomerulo-
nephritis
F Goodpasture’s syndrome
G IgA nephropathy
H Nephrotic syndrome
I Focal segmental glomerulo-
nephritis
A

A Nephritic syndrome

How well did you know this?
1
Not at all
2
3
4
5
Perfectly
37
Q
A 42-year-old man presents to accident and emergency with an episode of haemoptysis and haematuria. Blood tests reveal he is in acute renal failure. Once the patient is stable a renal biopsy demonstrates a crescent morphology on immunofluorescence.
A Nephritic syndrome
B Wegener’s granulomatosis
C Membranous glomerulonephritis
D Acute tubular necrosis
E Minimal change glomerulo-
nephritis
F Goodpasture’s syndrome
G IgA nephropathy
H Nephrotic syndrome
I Focal segmental glomerulo-
nephritis
A

F Goodpasture’s syndrome

How well did you know this?
1
Not at all
2
3
4
5
Perfectly
38
Q
A 64-year-old man on the Care of the Elderly ward is found to be in acute renal failure secondary to statin-related rhabdomyolysis. Urinalysis reveals the pres- ence of ‘muddy’ casts.
A Nephritic syndrome
B Wegener’s granulomatosis
C Membranous glomerulonephritis
D Acute tubular necrosis
E Minimal change glomerulo-
nephritis
F Goodpasture’s syndrome
G IgA nephropathy
H Nephrotic syndrome
I Focal segmental glomerulo-
nephritis
A

D Acute tubular necrosis

How well did you know this?
1
Not at all
2
3
4
5
Perfectly
39
Q
An 8-year-old girl presents to accident and emergency with frank haematuria. Her parents state that she had just recovered from a throat infection 2 days previously.
A Nephritic syndrome
B Wegener’s granulomatosis
C Membranous glomerulonephritis
D Acute tubular necrosis
E Minimal change glomerulo-
nephritis
F Goodpasture’s syndrome
G IgA nephropathy
H Nephrotic syndrome
I Focal segmental glomerulo-
nephritis
A

G IgA nephropathy

How well did you know this?
1
Not at all
2
3
4
5
Perfectly
40
Q
A 62-year-old woman on the Care of the Elderly ward is found to have new onset ankle swelling. A urine dipstick demonstrates proteinuria and the only blood abnormality is a low albumin level.
A Nephritic syndrome
B Wegener’s granulomatosis
C Membranous glomerulonephritis
D Acute tubular necrosis
E Minimal change glomerulo-
nephritis
F Goodpasture’s syndrome
G IgA nephropathy
H Nephrotic syndrome
I Focal segmental glomerulo-
nephritis
A

H Nephrotic syndrome

How well did you know this?
1
Not at all
2
3
4
5
Perfectly
41
Q
A 55-year-old parous woman presents to her GP with a 2-week history of green discharge from her right nipple.
A Mastitis
B Phylloides tumour
C Fibroadenoma
D Duct ectasia
E Ductal carcinoma in situ
F Gynaecomastia
G Fibrocystic disease
H Fat necrosis
I Infiltrating ductal carcinoma
A

D Duct ectasia

How well did you know this?
1
Not at all
2
3
4
5
Perfectly
42
Q
A 35-year-old woman presents to her GP with a soft 3 cm mobile mass in her left breast. The patient suggests the size of the lump fluctuates with her men- strual cycle.
A Mastitis
B Phylloides tumour
C Fibroadenoma
D Duct ectasia
E Ductal carcinoma in situ
F Gynaecomastia
G Fibrocystic disease
H Fat necrosis
I Infiltrating ductal carcinoma
A

C Fibroadenoma

How well did you know this?
1
Not at all
2
3
4
5
Perfectly
43
Q
A 54-year-old woman presents to her GP with a single lump in her left breast. A mammogram reveals a focal area of calcification.
A Mastitis
B Phylloides tumour
C Fibroadenoma
D Duct ectasia
E Ductal carcinoma in situ
F Gynaecomastia
G Fibrocystic disease
H Fat necrosis
I Infiltrating ductal carcinoma
A

E Ductal carcinoma in situ

How well did you know this?
1
Not at all
2
3
4
5
Perfectly
44
Q
A 60-year-old woman presents to her GP with a 5.5 cm mobile lump in her right breast. Biopsy reveals an ‘artichoke-like’ appearance.
A Mastitis
B Phylloides tumour
C Fibroadenoma
D Duct ectasia
E Ductal carcinoma in situ
F Gynaecomastia
G Fibrocystic disease
H Fat necrosis
I Infiltrating ductal carcinoma
A

B Phylloides tumour

How well did you know this?
1
Not at all
2
3
4
5
Perfectly
45
Q
A 58-year-old woman presents to her GP with a painful lump in her right breast. On examination there is also evidence of peau d’orange.
A Mastitis
B Phylloides tumour
C Fibroadenoma
D Duct ectasia
E Ductal carcinoma in situ
F Gynaecomastia
G Fibrocystic disease
H Fat necrosis
I Infiltrating ductal carcinoma
A

I Infiltrating ductal carcinoma

How well did you know this?
1
Not at all
2
3
4
5
Perfectly
46
Q
A 35-year-old man with pain and difficulty bending his left knee. X-ray reveals many lytic lesions in the epiphysis of the patient’s left knee.
A Osteoporosis
B Fibrous dysplasia
C Paget’s disease
D Osteomalacia
E Osteochondroma
F Osteoid osteoma
G Renal osteodytrophy 
H Enchondroma
I Giant cell tumour
A

I Giant cell tumour

How well did you know this?
1
Not at all
2
3
4
5
Perfectly
47
Q
A 38-year-old woman presents to her GP with generalized bone pain. X-ray reveals areas of pseudofracture, especially in the ribs.
A Osteoporosis
B Fibrous dysplasia
C Paget’s disease
D Osteomalacia
E Osteochondroma
F Osteoid osteoma
G Renal osteodytrophy 
H Enchondroma
I Giant cell tumour
A

D Osteomalacia

How well did you know this?
1
Not at all
2
3
4
5
Perfectly
48
Q
A 65-year-old woman is referred to the rheumatologist after suffering recurrent falls. Blood tests are all unremarkable but a DEXA scan reveals a T-score of 2.8.
A Osteoporosis
B Fibrous dysplasia
C Paget’s disease
D Osteomalacia
E Osteochondroma
F Osteoid osteoma
G Renal osteodytrophy 
H Enchondroma
I Giant cell tumour
A

A Osteoporosis

How well did you know this?
1
Not at all
2
3
4
5
Perfectly
49
Q
An 8-year-old boy has been diagnosed with precocious puberty. A routine examination by the paediatrician reveals café-au-lait spots on the child’s back. The boy has had numerous fractures of his femur and tibia bilaterally after falls.
A Osteoporosis
B Fibrous dysplasia
C Paget’s disease
D Osteomalacia
E Osteochondroma
F Osteoid osteoma
G Renal osteodytrophy 
H Enchondroma
I Giant cell tumour
A

B Fibrous dysplasia

How well did you know this?
1
Not at all
2
3
4
5
Perfectly
50
Q
A 50-year-old man presents to his GP with pain in his arms and legs. The patient also complains of shooting pains down his left leg as well as worsening shortness of breath.
A Osteoporosis
B Fibrous dysplasia
C Paget’s disease
D Osteomalacia
E Osteochondroma
F Osteoid osteoma
G Renal osteodytrophy 
H Enchondroma
I Giant cell tumour
A

C Paget’s disease

How well did you know this?
1
Not at all
2
3
4
5
Perfectly
51
Q
A 65-year-old patient with advanced breast malignancy and a history of multi- ple systemic emboli suffers a stroke. On examination, there are no cardiac mur- murs but an echocardiogram reveals small bland vegetations on the mitral valve. Blood cultures are negative. What is the most likely diagnosis?
A Infective endocarditis
B Acute rheumatic fever
C Non-bacterial thrombotic endocarditis
D Chronic rheumatic valvular disease
E Libman–Sacks endocarditis
A

C Non-bacterial thrombotic endocarditis

When I google- Libman Sacks endocarditis is the same as non bacterial thrombotic endocarditis….So i guess either is correct?

How well did you know this?
1
Not at all
2
3
4
5
Perfectly
52
Q
A 41-year-old man presents with severe central chest pain which he describes as ‘tearing’ in nature and radiating to the back. He is tall, with long limbs and long thin fingers. He also has an aortic regurgitation murmur. Histologically there is cystic medial necrosis in the aortic wall. In which syndrome are these findings most likely?
A Ortner’s syndrome
B Ehlers–Danlos syndrome
C Down syndrome
D Turner syndrome
E Marfan syndrome
A

E Marfan syndrome

How well did you know this?
1
Not at all
2
3
4
5
Perfectly
53
Q

A 57-year-old overweight patient suffers an acute myocardial infarction and subsequently dies. A post-morterm examination of the infarcted area shows extensive cell infiltration including polymorphs and macrophages. There is also extensive debris post necrosis and the cytoplasm is homogeneous making it difficult to see the outlines of the myocardial fibres. There is no evidence of collagenization or a scar. How long after the initial attack did the patient die?
A At the time of the attack (0–6 hours)
B Hours after the attack (6–24 hours)
C Days after the attack (1–4 days)
D Within the first 2 weeks of the attack (4–14 days)
E Weeks and months after (14 days +)

A

C Days after the attack (1–4 days)

<6hours = normal histology
6-24 hours = loss of nuclei, homogenous cytoplasm, necrotic cell death
1-4 days = infiltration of polymorphs then macrophages
5-10 days = removal or debris
1-2 weeks = granulation tissue, new blood vessels, myofibroblasts, collagen synthesis
Weeks-months = strengthening, decellularising scar

How well did you know this?
1
Not at all
2
3
4
5
Perfectly
54
Q
A 35-year-old woman presents to accident and emergency with nausea, severe malaise, swelling and stiffness of the fingers. On examination, her blood pressure is 155/95mmHg and she has Raynaud’s phenomenon. Blood tests reveal positive anti-topoisomerase antibodies and deranged serum creatinine and urea. A biopsy result of her small arteries reveals an onion skin appearance. What is the most likely diagnosis?
A Systemic lupus erythematosus
B Diffuse scleroderma
C Kawasaki’s disease
D Polyarteritis nodosa
E Limited scleroderma/CREST
A

B Diffuse scleroderma

How well did you know this?
1
Not at all
2
3
4
5
Perfectly
55
Q
A 46-year-old woman presents with gradual muscle weakness in her neck and upper arms over the past 3 weeks. She is also said to have a purple ‘heliotrope’ rash on her upper eyelids, an erythematous scaling rash on her face and red patches on the knees. She has also experienced some weight loss. Blood tests reveal elevated skeletal muscle enzymes but electromyogram results were nega- tive. What is the most likely diagnosis?
A Polymyositis
B Henoch–Schönlein purpura
C Dermatomyositis
D Kawasaki disease
E Sarcoidosis
A

C Dermatomyositis

How well did you know this?
1
Not at all
2
3
4
5
Perfectly
56
Q
A 43-year-old man presents with a rest tremor, slowness of voluntary movement and rigidity. It is reported that he has a mutation of the alpha-synuclein protein and he is free of Lewy bodies on histological examination. What is the most likely diagnosis?
A Familial Parkinson’s disease
B Alzheimer’s disease
C Multiple system atrophy
D Multiple sclerosis
E Idiopathic Parkinson’s disease
A

A Familial Parkinson’s disease

How well did you know this?
1
Not at all
2
3
4
5
Perfectly
57
Q
The activity of the plaques in a 25-year-old multiple sclerosis patient is described with the presence of oedema and macrophages, and some myelin breakdown. Which ICDNS (International Classification of Diseases of the Nervous System) plaque type classification best fits the description?
A Acute plaque
B Early chronic active plaque
C Late chronic active plaque
D Chronic inactive plaque
E Shadow plaque
A

B Early chronic active plaque

Acute plaque: Minor changes (e.g. oedema) and often difficult to recognize
Early chronic active plaque: Oedema and macrophages, indicative of an inflammatory disorder of the central nervous system, with some myelin breakdown. Reactive astrocytosis is present
Late chronic active plaque: Complete loss of myelin. Some macrophages will contain myelin debris and there will be often very mild perivascular inflammation at this stage with enlarged perivascular spaces
Chronic inactive plaque: Complete loss of myelin with the absence of macrophages
Shadow plaque: Nearly complete remyelination as a thin myelin with some scattered macrophages and a mild microglial up-regulation.

How well did you know this?
1
Not at all
2
3
4
5
Perfectly
58
Q
A 72-year-old woman is diagnosed with a disease that accounts for 50–75 per cent of all cases of dementia. The four characteristic pathological features for her diagnosis are severe brain atrophy, loss of neurons, senile plaques and neuro- fibllirary tangles. What is the most likely diagnosis?
A Huntington’s disease
B Alzheimer’s disease
C Multiple system atrophy
D Dementia with Lewy bodies
E Parkinson’s disease
A

B Alzheimer’s disease

How well did you know this?
1
Not at all
2
3
4
5
Perfectly
59
Q
A 32-year-old man has a past medical history of severe gastro-oesophageal reflux disease. His most recent oesophageal biopsy shows a columnar epithelium with goblet cells suggestive of a diagnosis of Barrett’s oesophagus. What form of cell change is this also known as?
A Anaplasia
B Hyperplasia
C Metaplasia
D Dysplasia
E Neoplasia
A

C Metaplasia

How well did you know this?
1
Not at all
2
3
4
5
Perfectly
60
Q

A 38-year-old man is a known gastritis patient. The most recent endoscopy and biopsy has detected that the area most severely affected is the pyloric antrum. He also has susceptibility for developing a gastric MALT lymphoma in the future. What is the most likely diagnosis?
A Menetrier’s disease (hyperplastic hypersecretory gastropathy)
B Acute gastritis
C Helicobacter-associated chronic gastritis
D Autoimmune chronic gastritis
E Reactive/reflux chronic gastritis

A

C Helicobacter-associated chronic gastritis

How well did you know this?
1
Not at all
2
3
4
5
Perfectly
61
Q
A 50-year-old known alcoholic man has persistent severe epigastric pain radiat- ing to the back and has experienced weight loss of 5kg in 2 months. On initial presentation, the patient is not jaundiced. On contrast enhanced CT scan there are multiple calcific densities along the line of the main pancreatic duct. On histological examination, there is evidence of parenchymal fibrosis and large ducts containing insipissated secretions. What is the most likely diagnosis?
A Chronic pancreatitis
B Carcinoma in the head of the pancreas
C Diabetes mellitus type 2
D Acute pancreatitis
E Pseudocysts
A

A. Chronic pancreatitis

How well did you know this?
1
Not at all
2
3
4
5
Perfectly
62
Q
A 22-year-old man presents with polyuria and polydipsia. His fasting plasma glucose is 7.3mmol/L. He is Glutamic Acid Decarboxylase (GAD) antibody positive. What is the most likely diagnosis?
A Diabetes mellitus type 1
B Diabetes insipidus
C Psychogenic polydipsia
D Diabetes mellitus type 2
E Zollinger–Ellison syndrome
A

A Diabetes mellitus type 1

How well did you know this?
1
Not at all
2
3
4
5
Perfectly
63
Q
A 37-year-old man, while abroad, was involved in a road traffic accident and required a blood transfusion. He had an episode of acute hepatitis with the con- traction of a DNA virus of the Hepadna group. There is a small chance this may progress to chronic hepatitis. What is the most likely viral hepatitis type?
A Hepatitis A
B Hepatitis B
C Hepatitis C
D Hepatitis D
E Hepatitis E
A

B Hepatitis B

How well did you know this?
1
Not at all
2
3
4
5
Perfectly
64
Q
A 42-year-old woman, who has a history of joint and skin symptoms, presents with jaundice. Anti-mitochondrial antibodies are present and histologically there is evidence of a progressive, chronic granulomatous inflammation of the bile duct. What is the most likely diagnosis?
A Primary sclerosing cholangitis
B Autoimmune hepatitis
C Primary biliary cirrhosis
D   -1 Antitrypsin deficiency
E Alcoholic liver disease
A

C Primary biliary cirrhosis

How well did you know this?
1
Not at all
2
3
4
5
Perfectly
65
Q

A 23-year-old patient has an autosomal recessive disorder. The patient has demonstrated parkinsonian symptoms such as a hand tremor and has developed chronic hepatitis. On examination, he is found to have Kayser–Fleischer
rings. Blood levels of serum ceruloplasmin are low. What is the most likely diagnosis?
A Wilson’s disease
B Genetic haemochromatosis
C -1 Antitrypsin deficiency
D Reye’s syndrome
E Budd–Chiari syndrome

A

A Wilson’s disease

How well did you know this?
1
Not at all
2
3
4
5
Perfectly
66
Q
An 8-year-old Down syndrome boy presents with constipation, distended abdo- men, vomiting and overflow diarrhoea. The cause is believed to be absence of ganglion cells in the myenteric plexus causing the failure of the dilation of the distal colon. What is the most likely diagnosis?
A Stenosis
B Hirschsprung’s disease
C Atresia
D Intussusception
E Volvulus
A

B Hirschsprung’s disease

How well did you know this?
1
Not at all
2
3
4
5
Perfectly
67
Q
A 25-year-old white man is experiencing bloody diarrhoea and mucous discharge. Macroscopic analysis shows abnormality in the colon and rectum only and is continuous with a normal bowel wall thickness. The pattern of inflammation isconfined to the mucosa of the bowel wall and no evidence of granulomas exists. What is the most likely diagnosis?
A Crohn’s disease
B Ulcerative colitis
C Ischaemic colitis
D Pseudomembranous colitis
E Viral gastroenteritis
A

B Ulcerative colitis

How well did you know this?
1
Not at all
2
3
4
5
Perfectly
68
Q

A 39-year-old man is diagnosed with a colon cancer proximal to the splenic flexure that is poorly differentiated and highly aggressive. There are no associ- ated adenomata. It is an autosomal dominant condition that involves gene muta- tions of DNA mismatch repair genes. What is the most likely diagnosis?
A Familial adenomatous polyposis
B Gardner’s syndrome
C Colorectal carcinoma
D Hereditary non-polyposis colorectal cancer
E Hamartomatous polyps

A

D Hereditary non-polyposis colorectal cancer

How well did you know this?
1
Not at all
2
3
4
5
Perfectly
69
Q
A 25-year-old woman presents to clinic with an inability to conceive and a past history of Chlamydia trachomatis infection. On ultrasonography, she is diagnosed with hydrosalpinx. Hydrosalpinx is the most likely complication of which of the below options?
A Endometriosis
B Adenomyosis
C Cervical intraepithelial neoplasia
D Salpingitis
E Human papillomavirus
A

D Salpingitis

How well did you know this?
1
Not at all
2
3
4
5
Perfectly
70
Q

A 42-year-old Afro-Caribbean woman is nulliparous and trying to conceive. She has been experiencing dysmenorrhoea. Ultrasound scan shows multiple rounded nodules within the myometrium. What is the most likely diagnosis?
A Cervical intraepithelial neoplasia
B Vulval carcinoma
C Leiomyoma
D Endometrial carcinoma
E CGIN (endocervical glandular dysplasia)

A

C Leiomyoma

How well did you know this?
1
Not at all
2
3
4
5
Perfectly
71
Q
A 20-year-old woman presents to accident and emergency with a distended abdomen resembling a pregnancy. She later develops acute onset of severe abdominal pain. An ultrasound identified a mass in her right ovary. Her abdomen is rigid and she is admitted for emergency surgery. It is believed that three embryonic germ cell layers are present. What is the most likely diagnosis?
A Teratoma of the ovary
B Serous tumour of the ovary
C Mucinous tumour of the ovary
D Endometrioid tumour of the ovary
E Clear cell carcinoma
A

A Teratoma of the ovary

How well did you know this?
1
Not at all
2
3
4
5
Perfectly
72
Q

A 32-year-old woman presents with generalized fatigue. Full blood count shows a reduced haemoglobin level and reduced mean corpuscular volume. A peripheral blood film has revealed iron deficiency anaemia. What features are most likely to be seen on her peripheral blood film?
A Hypochromic and microcytic red blood cells with anisopoikilocytosis and acanthocytes
B Hypochromic and microcytic red blood cells with hypersegmented neutrophils
C Hypochromic and microcytic red blood cells with anisopoikilocytosis and no evidence of basophilic stippling
D Hypochromic and microcytic red blood cells with Howell–Jolly bodies and basophilic stippling
E Hypochromic and macrocytic red blood cells with target cells, acanthocytes and Howell–Jolly bodies

A

C Hypochromic and microcytic red blood cells with anisopoikilocytosis and no evidence of basophilic stippling

How well did you know this?
1
Not at all
2
3
4
5
Perfectly
73
Q

A 26-year-old woman presents with fatigue ‘all the time’. She has a family his- tory of coeliac disease and blood tests reveal hypochromic, microcytic anaemia. She is referred to the gastroenterology clinic for tests. The gold standard investi- gation is the duodenal biopsy, which is carried out after positive serological test- ing. Which current serological testing and histopathology findings in the options below are most consistent with a coeliac disease diagnosis?
A Anti-reticulin antibodies only/villous atrophy, crypt hyperplasia, increased intraepithelial lymphocytes
B Anti-gliadin antibodies only/no villous atrophy, crypt hyperplasia, decreased intraepithelial lymphocytes
C Anti-endomysial antibodies only/villous atrophy, crypt hyperplasia, increased intraepithelial lymphocytes
D Anti-endomysial antibodies and anti-tissue transglutaminase antibodies/ villous atrophy, crypt hyperplasia, increased intraepithelial lymphocytes
E Anti-endomysial antibodies and anti-tissue transglutaminase antibodies/villous atrophy, no evidence of crypt hyperplasia, increased intraepithelial lymphocytes

A

D Anti-endomysial antibodies and anti-tissue transglutaminase antibodies/ villous atrophy, crypt hyperplasia, increased intraepithelial lymphocytes

How well did you know this?
1
Not at all
2
3
4
5
Perfectly
74
Q

A 27-year-old woman has developed pain in her right proximal femur. She has
a history of intermittent hip pain since childhood. An X-ray has demonstrated a ‘soap bubble’ appearance indicative of osteolysis and a characteristic shepherd’s crook deformity. The biopsy would show irregular trabeculae of woven bone said to resemble Chinese letters. What is the most likely diagnosis?
A Non-ossifying fibroma
B Fibrous dysplasia
C Giant cell reparative granuloma
D Ossifying fibroma
E Simple bone cyst

A

B Fibrous dysplasia

How well did you know this?
1
Not at all
2
3
4
5
Perfectly
75
Q
A 36-year-old man presents with swelling of his middle finger and subsequently a fracture. His X-ray shows cotton wool calcification and histopathology shows evidence of a tumour composed of benign hyaline cartilage. It is believed that he has only a very slight risk of malignant transformation. What is the most likely diagnosis?
A Osteochondroma
B Multiple myeloma
C Osteoid osteoma
D Giant cell tumour
E Enchondroma
A

E Enchondroma

How well did you know this?
1
Not at all
2
3
4
5
Perfectly
76
Q
An 18-year-old man presents with pain and a mass in his right knee. His X-ray shows an ill defined mass in the metaphyseal region of the distal femur that is sclerotic and lytic. There is also an elevated periosteum (known as a Codman’s tri- angle). Prognosis is said to be poor and the treatment required is multi-disciplinary involving intensive chemotherapy and surgery. In cytology, these tumour cells will be positive for alkaline phosphatase. What is the most likely diagnosis?
A Osteosarcoma
B Chondrosarcoma
C Fibrosarcoma
D Malignant fibrous histiocytoma
E Ewing’s sarcoma
A

A Osteosarcoma

How well did you know this?
1
Not at all
2
3
4
5
Perfectly
77
Q
A 25-year-old woman is due for her cervical smear test. Which method of cyto- pathology is going to be used?
A FNA
B Ultrasound guided FNA
C Washings
D Brushings
E Liquid based cytology
A

E Liquid based cytology

How well did you know this?
1
Not at all
2
3
4
5
Perfectly
78
Q
A 57-year-old man who is a heavy smoker presents to his GP with gradually worsening dyspnoea and cough productive of green sputum. On examination, he is cyanosed, tachypnoeic and wheezing. What is the most likely diagnosis?
A Chronic bronchitis
B Pulmonary embolus
C Asthma
D Bronchiolitis
E Emphysema
A

A Chronic bronchitis

How well did you know this?
1
Not at all
2
3
4
5
Perfectly
79
Q
A 57-year-old man presents to accident and emergency with dyspnoea, fever, cough and purulent sputum. Histopathology confirms widespread fibrinosup- purative consolidation on the left lower lobe and the top differential diagnosis is lobar pneumonia. Which organism is the most likely cause?
A Streptococcus pneumoniae
B Staphylococcus aureus
C Haemophilus influenzae
D Streptococcus pyogenes
E Mycobacterium tuberculosis
A

A Streptococcus pneumoniae

How well did you know this?
1
Not at all
2
3
4
5
Perfectly
80
Q
A 27-year-old man with severe second degree burns is admitted to the ITU and develops severe shortness of breath and tachypnoea the next day. Diffuse alveolar damage is indicated in the histopathology report. What is the most likely diagnosis?
A Pulmonary oedema
B Acute respiratory distress syndrome
C Cryptogenic fibrosing alveolitis
D Bronchiectasis
E Chronic bronchitis
A

B Acute respiratory distress syndrome

How well did you know this?
1
Not at all
2
3
4
5
Perfectly
81
Q
A 47-year-old construction worker presents with a 6-month history of cough, haemoptysis and 5kg weight loss. He is a heavy smoker and a centrally located lesion is found on his chest X-ray. Histology showed keratinization and intercel- lular ‘prickles’. What is the most likely diagnosis?
A Tuberculosis
B Squamous cell carcinoma
C Mesothelioma
D Emphysema
E Large cell carcinoma
A

B Squamous cell carcinoma

How well did you know this?
1
Not at all
2
3
4
5
Perfectly
82
Q
A 55-year-old non-smoking woman presents to her GP with a 6-month history of cough, haemoptysis and 5kg weight loss. A chest X-ray showed the lesion is in the periphery and histopathology showed evidence of glandular differentiation and cytology showed mucin vacuoles. Mode of treatment most suitable is surgi- cal. What is the most likely diagnosis?
A Small cell carcinoma
B Adenocarcinoma
C Large cell carcinoma
D Sarcoidosis
E Pneuomoconiosis
A

B Adenocarcinoma

How well did you know this?
1
Not at all
2
3
4
5
Perfectly
83
Q
A 27-year-old man presents with fever, fatigue and a rash. He has also noted a few painful ulcers in his mouth. The rash is described as numerous round lesions about an inch in diameter on the face, trunk, arms and legs, diagnosed as erythema multiforme. What is the most likely diagnosis for this patient?
A Systemic lupus erythematosus
B Stevens–Johnson syndrome
C Pemphigoid
D Pityriasis rosea
E Contact dermatitis
A

B Stevens–Johnson syndrome

How well did you know this?
1
Not at all
2
3
4
5
Perfectly
84
Q
A 55-year-old Australian man presents with a flat black lesion on his back that appears asymmetrical with an irregular border and 6mm in diameter. Breslow’s depth is 0.4mm. What is the most likely diagnosis?
A Malignant melanoma
B Basal cell carcinoma
C Squamous cell carcinoma
D Keratoacanthoma
E Bowen’s disease
A

A Malignant melanoma

How well did you know this?
1
Not at all
2
3
4
5
Perfectly
85
Q
A 23-year-old Irish man presents with an itchy blistering eruption on his but- tocks and elbows. He also has diarrhoea and abdominal pain. Histopathology reveals papillary microabscesses and a neutrophilic infiltrate. He has a family history of gluten sensitivity. Which rash is most often associated with his presentation?
A Psoriasis
B Atopic eczema
C Dermatitis herpetiformis
D Lichen planus
E Seborrhoeic dermatitis
A

C Dermatitis herpetiformis

How well did you know this?
1
Not at all
2
3
4
5
Perfectly
86
Q
A 26-year-old man presents to accident and emergency having fallen off his skateboard and landed with a big impact on his right side. His X-ray shows a fracture in the midshaft of his right humerus that appears splintered although the soft tissue is intact. What type of fracture is this?
A Greenstick fracture
B Transverse fracture
C Compound fracture
D Impacted fracture
E Comminuted fracture
A

E Comminuted fracture

How well did you know this?
1
Not at all
2
3
4
5
Perfectly
87
Q
An 80-year-old woman presents complaining of pain on movement and stiff- ness after inactivity in her legs, most notably in her hips and knees. She also complains of pain in her hands and marked symmetrical swelling is noted in her distal interphalangeal joints. The X-ray of her right knee shows subchondral sclerosis, subchondral cyst formation, joint space narrowing and osteophytes. What is the most likely diagnosis?
A Osteoarthritis
B Rheumatoid arthritis
C Ankylosing spondylitis
D Psoriatic arthritis
E Osteoporosis
A

A Osteoarthritis

How well did you know this?
1
Not at all
2
3
4
5
Perfectly
88
Q
A 59-year-old man presents to accident and emergency with a painful, swollen and hot big toe. The joint aspirate shows negatively birefringent crystals under polarized red light. The crystals are needle shaped. What is the most likely diagnosis?
A Pseudogout
B Lyme disease
C Reiter’s sydrome
D Gout
E Osteomyelitis
A

D Gout

How well did you know this?
1
Not at all
2
3
4
5
Perfectly
89
Q
A 23-year-old man presents to accident and emergency with a 2-day history of left-sided loin pain, fever, rigors and vomiting. Urine analysis reveals microscop- ic haematuria and white cell casts. What is the most likely diagnosis?
A Cystitis
B Prostatitis
C Urolithiasis
D Acute pyelonephritis
E Hydronephrosis
A

D Acute pyelonephritis

How well did you know this?
1
Not at all
2
3
4
5
Perfectly
90
Q
A 4-year-old boy presents with a large abdominal mass and haematuria. His blood pressure is 165/120mmHg. The mass has a large necrotic solid tumour with extrarenal invasion. Microscopically, there are immature-looking glomerular structures. Aggressive therapy with surgery, chemotherapy and radiotherapy is indicated. What is the most likely diagnosis?
A Teratoma
B Wilm’s tumour
C Oncocytoma
D Spermatocytic seminoma
E Bowen’s disease
A

B Wilm’s tumour

How well did you know this?
1
Not at all
2
3
4
5
Perfectly
91
Q

A 79-year-old man has hesitancy and terminal dribbling urinary symptoms secondary to a tumour growth. No other symptoms are present. On rectal exami- nation, the prostate is reported as hard and craggy. The patient has been given
a Gleason’s score of 8; a primary grade of 3, describing that the tissue has rec- ognizable glands and these cells are beginning to invade the surrounding tissue. There is also a secondary grade of pattern 5 suggesting poorly differentiated cells. What is the most likely diagnosis?
A Prostatic adenocarcinoma
B Seminoma
C Prostatic intraepithelial neoplasia
D Benign prostatic hyperplasia
E Transitional cell carcinoma

A

A Prostatic adenocarcinoma

How well did you know this?
1
Not at all
2
3
4
5
Perfectly
92
Q
A 27-year-old lactating mother presents with a painful red left breast. On closer examination, there are cracks and fissures on the left nipple. What is the most likely diagnosis?
A Fat necrosis
B Acute mastitis
C Duct ectasia
D Simple fibrocystic change
E Epithelial hyperplasia
A

B Acute mastitis

How well did you know this?
1
Not at all
2
3
4
5
Perfectly
93
Q
A 56-year-old woman presents with blood stained nipple discharge and a soli- tary mass located just superior to the nipple in her left breast. A histopathology analysis shows that a papillary mass is lined by epithelium and myoepithelium. It is believed that there is no increased risk of malignancy. What is the most likely diagnosis?
A Intraductal papilloma
B Phylloides tumour
C Fibroadenoma
D Radial scar
E Ductal carcinoma in situ
A

A Intraductal papilloma

How well did you know this?
1
Not at all
2
3
4
5
Perfectly
94
Q

A 53-year-old overweight woman with a positive family history of breast cancer attends her appointment for the NHS Breast Screening Programme. She is oneof the 5 per cent of women who have an abnormal mammogram and are called for a core biopsy. She has been given a B code of B5b. What is the most likely diagnosis?
A Benign abnormality
B Lesion of uncertain malignant potential
C Ductal carcinoma in situ
D Invasive carcinoma
E Suspicious of malignancy

A

D Invasive carcinoma

How well did you know this?
1
Not at all
2
3
4
5
Perfectly
95
Q
A 44-year-old man has developed end stage renal failure over the past 5 years with numerous episodes of macroscopic haematuria. He was asymptomatic previ- ously. Ultrasound scan has shown numerous asymmetrical large cysts bilaterally. The patient’s mother had a similar condition. What is the most likely diagnosis?
A Acquired cystic disease
B Medullary sponge disease
C Adult polycystic kidney disease
D Cystic renal dysplasia
E Simple renal cysts
A

C Adult polycystic kidney disease

How well did you know this?
1
Not at all
2
3
4
5
Perfectly
96
Q

A 48-year-old man presents with oliguria and a vasculitic rash on his legs. Investigations indicate that he has a reduced glomerular filtration rate and uri- nalysis finds urine casts containing red and white blood cells. Histopathology shows scanty deposits of immunoglobulins and complement present with associ- ated anti-neutrophil cytoplasm antibodies (ANCA). What is the most likely diagnosis?
A IgA nephropathy
B Thrombotic microangiopathy
C Anti-GBM crescentic glomerulonephritis disease
D Pauci-immune crescentic glomerulonephritis disease
E Amyloidosis

A

D Pauci-immune crescentic

A type of acute crescentic glomerulonephritis with the presence of ANCA. It is rapidly progressive

How well did you know this?
1
Not at all
2
3
4
5
Perfectly
97
Q
A 35-year-old oedematous woman is found to have urinary protein loss of 5.1g daily. Further tests show a low albumin level and significant interference with podocyte function. No glomerular crescents were detected. What is the most likely diagnosis?
A Acute glomerulonephritis
B Nephritic syndrome
C Nephrotic syndrome
D Acute tubular necrosis
E Acquired cystic disease
A

C Nephrotic syndrome

How well did you know this?
1
Not at all
2
3
4
5
Perfectly
98
Q
A 58-year-old woman is known to have a Berry aneurysm in the basilar artery. She develops sudden onset severe headache, nausea and loss of consciousness. There was evidence of a ‘warning bleed’ but no history of brain trauma. What is the most likely fatal diagnosis caused by the ruptured Berry aneurysm?
A Intracerebral haemorrhage
B Subarachnoid haemorrhage
C ‘Watershed’ strokes
D Transient ischaemic attack
E Tonsillar brain herniation
A

B Subarachnoid haemorrhage

How well did you know this?
1
Not at all
2
3
4
5
Perfectly
99
Q
A 9-year-old boy presents with fever, headache, stiff neck and altered men- tal state. His cerebrospinal fluid is turbid and contains mostly neutrophils. The meninges appear congested and there is purulent material in the subarachnoid space. What is the most likely causative organism?
A Coxsackie virus
B Treponema pallidum
C Staphylococcus aureus
D Streptococcus pneumoniae
E Haemophilus influenzae
A

D Streptococcus pneumoniae

How well did you know this?
1
Not at all
2
3
4
5
Perfectly
100
Q
A 22-year-old HIV-infected woman is pregnant with her second child. HIV has been transmitted perinatally to her first child. One of the most successful inter- ventions to reduce vertical transmission of HIV during pregnancy to less than 1 per cent is the use of combination anti-retroviral treatment, which ideally should reduce the viral load to?
A 50 copies/mL
B 800 copies/mL
C 1000 copies/mL
D 5000 copies/mL
E 10 000 copies/mL
A

A 50 copies/mL

How well did you know this?
1
Not at all
2
3
4
5
Perfectly
101
Q
A 70 year old lady presents to her GP complaining of tight chest pain, which radiated to her left arm and was relieved by rest. Her ECG revealed some st depression
A. Aortic Dissection
B. Coarctation of the Aorta
C. Dressler’s Syndrome
D. Restrictive cardiomyopathy
E. Pulmonary embolus
F. Decubitus Angina
G. Acute Coronary Syndrome
H. Transient Ischaemic Attack
I. Aortic Stenosis
J. Cerebrovascular Accident
K. Acute Myocardial Infarction
L. Mitral Stenosis
M. Stable Angina
N. Pericarditis
O. Ventricular Tachycardia
A

M. Stable Angina

How well did you know this?
1
Not at all
2
3
4
5
Perfectly
102
Q
A 50 year old male smoker, with a history of hypertension presents to the A and E department with continuous, central, crushing chest pain radiating to the left arm. The ECG showed st elevation.
A. Aortic Dissection
B. Coarctation of the Aorta
C. Dressler’s Syndrome
D. Restrictive cardiomyopathy
E. Pulmonary embolus
F. Decubitus Angina
G. Acute Coronary Syndrome
H. Transient Ischaemic Attack
I. Aortic Stenosis
J. Cerebrovascular Accident
K. Acute Myocardial Infarction
L. Mitral Stenosis
M. Stable Angina
N. Pericarditis
O. Ventricular Tachycardia
A

K. Acute Myocardial Infarction

How well did you know this?
1
Not at all
2
3
4
5
Perfectly
103
Q
A 63 year old obese, diabetic male presents to A and E with tight chest pain at rest, which radiated to the left arm and lasted for less than 20 minutes. The CK was not raised.
A. Aortic Dissection
B. Coarctation of the Aorta
C. Dressler’s Syndrome
D. Restrictive cardiomyopathy
E. Pulmonary embolus
F. Decubitus Angina
G. Acute Coronary Syndrome
H. Transient Ischaemic Attack
I. Aortic Stenosis
J. Cerebrovascular Accident
K. Acute Myocardial Infarction
L. Mitral Stenosis
M. Stable Angina
N. Pericarditis
O. Ventricular Tachycardia
A

G. Acute Coronary Syndrome

How well did you know this?
1
Not at all
2
3
4
5
Perfectly
104
Q
A 68 year old man presents with sudden onset chest pain, which radiated to the back. On examination the patient was shocked, with a hemiplegia and the chest X-ray showed mediastinal enlargement.
A. Aortic Dissection
B. Coarctation of the Aorta
C. Dressler’s Syndrome
D. Restrictive cardiomyopathy
E. Pulmonary embolus
F. Decubitus Angina
G. Acute Coronary Syndrome
H. Transient Ischaemic Attack
I. Aortic Stenosis
J. Cerebrovascular Accident
K. Acute Myocardial Infarction
L. Mitral Stenosis
M. Stable Angina
N. Pericarditis
O. Ventricular Tachycardia
A

A. Aortic Dissection

How well did you know this?
1
Not at all
2
3
4
5
Perfectly
105
Q
A 73 year old man with a known history of peripheral vascular disease presents to the A and E department with a sudden onset hemiplegia which resolved within 24 hours.
A. Aortic Dissection
B. Coarctation of the Aorta
C. Dressler’s Syndrome
D. Restrictive cardiomyopathy
E. Pulmonary embolus
F. Decubitus Angina
G. Acute Coronary Syndrome
H. Transient Ischaemic Attack
I. Aortic Stenosis
J. Cerebrovascular Accident
K. Acute Myocardial Infarction
L. Mitral Stenosis
M. Stable Angina
N. Pericarditis
O. Ventricular Tachycardia
A

H. Transient Ischaemic Attack

How well did you know this?
1
Not at all
2
3
4
5
Perfectly
106
Q
A 65 year old man is in hospital after suffering an acute myocardial infarction. The house officer hears a pansystolic murmur on auscultation.
A. Rheumatic fever
B. Hypertrophic cardiomyopathy
C. Infective endocarditis
D. Aortic regurgitation
E. Pericarditis
F. Myomalacia cordis
G. Dilated cardiomyopathy
H. Mitral regurgitation
I. Myxomatous mitral valve
J. Pericardial effusion
K. Aortic stenosis
L. Congenital septal defect
A

F. Myomalacia cordis

There is softening of dead muscle which eventually leads to rupture and death.

How well did you know this?
1
Not at all
2
3
4
5
Perfectly
107
Q
A 28 year old sportsman presents to A&amp;E with severe chest pain and breathlessness. He has a history of asthma. There is a systolic murmur on examination.
A. Rheumatic fever
B. Hypertrophic cardiomyopathy
C. Infective endocarditis
D. Aortic regurgitation
E. Pericarditis
F. Myomalacia cordis
G. Dilated cardiomyopathy
H. Mitral regurgitation
I. Myxomatous mitral valve
J. Pericardial effusion
K. Aortic stenosis
L. Congenital septal defect
A

B. Hypertrophic cardiomyopathy

How well did you know this?
1
Not at all
2
3
4
5
Perfectly
108
Q
39 year old lady suffers a sharp retrosternal chest pain which is worse on inspiration. The finding on auscultation is typical of this presentation.
A. Rheumatic fever
B. Hypertrophic cardiomyopathy
C. Infective endocarditis
D. Aortic regurgitation
E. Pericarditis
F. Myomalacia cordis
G. Dilated cardiomyopathy
H. Mitral regurgitation
I. Myxomatous mitral valve
J. Pericardial effusion
K. Aortic stenosis
L. Congenital septal defect
A

E. Pericarditis

How well did you know this?
1
Not at all
2
3
4
5
Perfectly
109
Q
A middle aged women is in hospital after fainting at the gym. She has a severe headache and feels generally unwell. There is a systolic ejection murmur on examination	
A. Rheumatic fever
B. Hypertrophic cardiomyopathy
C. Infective endocarditis
D. Aortic regurgitation
E. Pericarditis
F. Myomalacia cordis
G. Dilated cardiomyopathy
H. Mitral regurgitation
I. Myxomatous mitral valve
J. Pericardial effusion
K. Aortic stenosis
L. Congenital septal defect
A

K. Aortic stenosis

How well did you know this?
1
Not at all
2
3
4
5
Perfectly
110
Q
A 46 year old women presents to A&amp;E out of breath and with severe chest pain. On examination a mid systolic click late systolic murmur is revealed.
A. Rheumatic fever
B. Hypertrophic cardiomyopathy
C. Infective endocarditis
D. Aortic regurgitation
E. Pericarditis
F. Myomalacia cordis
G. Dilated cardiomyopathy
H. Mitral regurgitation
I. Myxomatous mitral valve
J. Pericardial effusion
K. Aortic stenosis
L. Congenital septal defect
A

I. Myxomatous mitral valve

Mitral valve prolapse

How well did you know this?
1
Not at all
2
3
4
5
Perfectly
111
Q
A 10 year old boy presents with skin rash and joint pain in his elbows and knees. His mother tells you that he recently had a sore throat. On examination he is found to have an ejection systolic murmur and a friction rub.
A. Aortic stenosis - Degenerative
B. Cardiomyopathy (Obliterative)
C. Chronic rheumatic valvular disease
D. Cardiac Failure
E. Acute rheumatic fever
F. Pericarditis
G. Subacute bacterial endocarditis
H. Cardiomyopathy (Hypertrophic)
I. Cardiomyopathy (Restrictive
J. Acute bacterial endocarditis
K. Non infective endocarditis
L. Cardiomyopathy (Dilated)
M. Aortic stenosis – acquired
A

E. Acute rheumatic fever

How well did you know this?
1
Not at all
2
3
4
5
Perfectly
112
Q
A 69 year old woman is suffering from sudden onset fever and malaise. There is no previous history of heart disease. Auscultation reveals a heart murmur. She later develops sepsis.
A. Aortic stenosis - Degenerative
B. Cardiomyopathy (Obliterative)
C. Chronic rheumatic valvular disease
D. Cardiac Failure
E. Acute rheumatic fever
F. Pericarditis
G. Subacute bacterial endocarditis
H. Cardiomyopathy (Hypertrophic)
I. Cardiomyopathy (Restrictive
J. Acute bacterial endocarditis
K. Non infective endocarditis
L. Cardiomyopathy (Dilated)
M. Aortic stenosis – acquired
A

J. Acute bacterial endocarditis

How well did you know this?
1
Not at all
2
3
4
5
Perfectly
113
Q
A 40 year old man presents with a sharp chest pain. He has a pericardial friction rub, diminished heart sounds and a raised JVP.	
A. Aortic stenosis - Degenerative
B. Cardiomyopathy (Obliterative)
C. Chronic rheumatic valvular disease
D. Cardiac Failure
E. Acute rheumatic fever
F. Pericarditis
G. Subacute bacterial endocarditis
H. Cardiomyopathy (Hypertrophic)
I. Cardiomyopathy (Restrictive
J. Acute bacterial endocarditis
K. Non infective endocarditis
L. Cardiomyopathy (Dilated)
M. Aortic stenosis – acquired
A

F. Pericarditis

How well did you know this?
1
Not at all
2
3
4
5
Perfectly
114
Q
A 25 year old man presents with palpitations. Chest X-ray shows an enlarged heart and echocardiogram shows thickening of the septum.
A. Aortic stenosis - Degenerative
B. Cardiomyopathy (Obliterative)
C. Chronic rheumatic valvular disease
D. Cardiac Failure
E. Acute rheumatic fever
F. Pericarditis
G. Subacute bacterial endocarditis
H. Cardiomyopathy (Hypertrophic)
I. Cardiomyopathy (Restrictive
J. Acute bacterial endocarditis
K. Non infective endocarditis
L. Cardiomyopathy (Dilated)
M. Aortic stenosis – acquired
A

H. Cardiomyopathy (Hypertrophic)

How well did you know this?
1
Not at all
2
3
4
5
Perfectly
115
Q
A 75 year old diabetic female with a history of 4 myocardial infarctions presents with shortness of breath and ankle swelling. She was found to have an enlarged liver and echocardiogram demonstrated a dilated heart.
A. Aortic stenosis - Degenerative
B. Cardiomyopathy (Obliterative)
C. Chronic rheumatic valvular disease
D. Cardiac Failure
E. Acute rheumatic fever
F. Pericarditis
G. Subacute bacterial endocarditis
H. Cardiomyopathy (Hypertrophic)
I. Cardiomyopathy (Restrictive
J. Acute bacterial endocarditis
K. Non infective endocarditis
L. Cardiomyopathy (Dilated)
M. Aortic stenosis – acquired
A

D. Cardiac Failure

How well did you know this?
1
Not at all
2
3
4
5
Perfectly
116
Q
A 19 year old American student with bronchiectasis is on inhaled tobramycin for chronic Pseudomonal infection. The mutation delta F508 is identified.
A. Chronic alcoholic pancreatitis
B. Renal tubular acidosis
C. Insulinoma
D. Gallstones
E. Hypercalcaemia
F. Vibrio cholerae infection
G. Carcinoma head of the pancreas
H. Gallstone pancreatitis
I. Pseudocysts
J. VIPoma (Werner Morrison syndrome)
K. Iatrogenic pancreatitis
L. Cystic fibrosis
M. Carcinoma tail of the pancreas
N. Haemochromatosis
A

L. Cystic fibrosis

How well did you know this?
1
Not at all
2
3
4
5
Perfectly
117
Q
A 68 year old smoker presents with jaundice and worsening abdominal and back pain. Scratch marks are seen on his arms and legs. He has lost 5kg in 2 months. Ultrasound shows dilated intrahepatic bile ducts
A. Chronic alcoholic pancreatitis
B. Renal tubular acidosis
C. Insulinoma
D. Gallstones
E. Hypercalcaemia
F. Vibrio cholerae infection
G. Carcinoma head of the pancreas
H. Gallstone pancreatitis
I. Pseudocysts
J. VIPoma (Werner Morrison syndrome)
K. Iatrogenic pancreatitis
L. Cystic fibrosis
M. Carcinoma tail of the pancreas
N. Haemochromatosis
A

G. Carcinoma head of the pancreas

How well did you know this?
1
Not at all
2
3
4
5
Perfectly
118
Q
A 39 year old Nepalese man presents with severe watery diarrhoea. He is found to have hypokalaemia and, surprisingly, a metabolic acidosis. A RUQ mass is detected by contrast-enhanced spiral CT scanning. Stool bicarb is high and urine anion gap is negative.
A. Chronic alcoholic pancreatitis
B. Renal tubular acidosis
C. Insulinoma
D. Gallstones
E. Hypercalcaemia
F. Vibrio cholerae infection
G. Carcinoma head of the pancreas
H. Gallstone pancreatitis
I. Pseudocysts
J. VIPoma (Werner Morrison syndrome)
K. Iatrogenic pancreatitis
L. Cystic fibrosis
M. Carcinoma tail of the pancreas
N. Haemochromatosis
A

J. VIPoma (Werner Morrison syndrome)

A vipoma is a non-beta pancreatic islet cell tumor secreting vasoactive intestinal peptide (VIP), resulting in a syndrome of watery diarrhea, hypokalemia, and achlorhydria (WDHA syndrome).

How well did you know this?
1
Not at all
2
3
4
5
Perfectly
119
Q
A 59 year old widow complains of persistent back pain, loss of appetite and that she has dropped from dress size 18 to a size 14 in just 2 months. She was recently diagnosed with diabetes. A large central mass is palpable as well hepatosplenomegaly.
A. Chronic alcoholic pancreatitis
B. Renal tubular acidosis
C. Insulinoma
D. Gallstones
E. Hypercalcaemia
F. Vibrio cholerae infection
G. Carcinoma head of the pancreas
H. Gallstone pancreatitis
I. Pseudocysts
J. VIPoma (Werner Morrison syndrome)
K. Iatrogenic pancreatitis
L. Cystic fibrosis
M. Carcinoma tail of the pancreas
N. Haemochromatosis
A

M. Carcinoma tail of the pancreas

How well did you know this?
1
Not at all
2
3
4
5
Perfectly
120
Q
A 47 year old lecturer is referred to hospital clinic from his GP with worsening abdominal pain. He has a poor diet and weight loss. He has previously been prescribed Thiamine.
A. Chronic alcoholic pancreatitis
B. Renal tubular acidosis
C. Insulinoma
D. Gallstones
E. Hypercalcaemia
F. Vibrio cholerae infection
G. Carcinoma head of the pancreas
H. Gallstone pancreatitis
I. Pseudocysts
J. VIPoma (Werner Morrison syndrome)
K. Iatrogenic pancreatitis
L. Cystic fibrosis
M. Carcinoma tail of the pancreas
N. Haemochromatosis
A

A. Chronic alcoholic pancreatitis

How well did you know this?
1
Not at all
2
3
4
5
Perfectly
121
Q
65 year old female with a large, cystic mass on tail of pancreas imaged using computed tomography. Further cytology reported the presence of epithelium
A. Scorpion Sting
B. Jaundice
C. Trousseau’s Syndrome
D. Thrombophlebitis
E. Agenesis
F. Alcoholism
G. Type 1 Diabetes
H. Pseudocyst
I. Pancreatitis
J. Cystadenoma
K. Whipples' resection
L. Cystic Fibrosis
M. Carcinoma of the Pancreas
N. Pancreas Divisum
O. Hyperlipidaemia
P. Gall Bladder
A

J. Cystadenoma

How well did you know this?
1
Not at all
2
3
4
5
Perfectly
122
Q
55 year old, diabetic, afro-Caribbean male presents with weight loss, poor diet and a gnawing pain in his back, which is sometimes felt ‘under his chest’
A. Scorpion Sting
B. Jaundice
C. Trousseau’s Syndrome
D. Thrombophlebitis
E. Agenesis
F. Alcoholism
G. Type 1 Diabetes
H. Pseudocyst
I. Pancreatitis
J. Cystadenoma
K. Whipples' resection
L. Cystic Fibrosis
M. Carcinoma of the Pancreas
N. Pancreas Divisum
O. Hyperlipidaemia
P. Gall Bladder
A

M. Carcinoma of the Pancreas

How well did you know this?
1
Not at all
2
3
4
5
Perfectly
123
Q
The commonest cause of acute pancreatitis in the UK
A. Scorpion Sting
B. Jaundice
C. Trousseau’s Syndrome
D. Thrombophlebitis
E. Agenesis
F. Alcoholism
G. Type 1 Diabetes
H. Pseudocyst
I. Pancreatitis
J. Cystadenoma
K. Whipples' resection
L. Cystic Fibrosis
M. Carcinoma of the Pancreas
N. Pancreas Divisum
O. Hyperlipidaemia
P. Gall Bladder
A

F. Alcoholism

How well did you know this?
1
Not at all
2
3
4
5
Perfectly
124
Q
Inflammatory condition of the exocrine pancreas that results in injury to acinar cells.
A. Scorpion Sting
B. Jaundice
C. Trousseau’s Syndrome
D. Thrombophlebitis
E. Agenesis
F. Alcoholism
G. Type 1 Diabetes
H. Pseudocyst
I. Pancreatitis
J. Cystadenoma
K. Whipples' resection
L. Cystic Fibrosis
M. Carcinoma of the Pancreas
N. Pancreas Divisum
O. Hyperlipidaemia
P. Gall Bladder
A

I. Pancreatitis

How well did you know this?
1
Not at all
2
3
4
5
Perfectly
125
Q
ERCP finding due to incomplete fusing of pancreatic buds
A. Scorpion Sting
B. Jaundice
C. Trousseau’s Syndrome
D. Thrombophlebitis
E. Agenesis
F. Alcoholism
G. Type 1 Diabetes
H. Pseudocyst
I. Pancreatitis
J. Cystadenoma
K. Whipples' resection
L. Cystic Fibrosis
M. Carcinoma of the Pancreas
N. Pancreas Divisum
O. Hyperlipidaemia
P. Gall Bladder
A

N. Pancreas Divisum

How well did you know this?
1
Not at all
2
3
4
5
Perfectly
126
Q
A condition in which the normal squamous epithelial lining of the oesophagus is replaced by columnar epithelium because of damage caused by gastro –oesophageal reflux or oesophagitis. The condition may be associated with an ulcer, and the epithelium has an abnormally high likelihood of undergoing malignant change.
A. Barrett’s oesophagus
B. Pernicious anaemia
C. Squamous carcinoma
D. Gastric ulcer
E. Gastric cancer
F. Intestinal metaplasia
G. Reflux oesophagitis
H. Helicobacter pylori
I. Oesophageal varices
J. Adenocarcinoma
K. Duodenal ulceration
L. Campylobacter jejuni
A

A. Barrett’s oesophagus

How well did you know this?
1
Not at all
2
3
4
5
Perfectly
127
Q
Caused by the action of acid and pepsin on the duodenal mucosa. Associated with increased output of stomach acid. Symptoms include pain in the upper abdomen, especially when the stomach is empty.
A. Barrett’s oesophagus
B. Pernicious anaemia
C. Squamous carcinoma
D. Gastric ulcer
E. Gastric cancer
F. Intestinal metaplasia
G. Reflux oesophagitis
H. Helicobacter pylori
I. Oesophageal varices
J. Adenocarcinoma
K. Duodenal ulceration
L. Campylobacter jejuni
A

K. Duodenal ulceration

How well did you know this?
1
Not at all
2
3
4
5
Perfectly
128
Q
The result of failure to produce intrinsic factor, and the subsequent reduction in the absorption of B12 from the bowel. Characterised by the defective production of red blood cells and the presence of megaloblasts in the bone marrow.
A. Barrett’s oesophagus
B. Pernicious anaemia
C. Squamous carcinoma
D. Gastric ulcer
E. Gastric cancer
F. Intestinal metaplasia
G. Reflux oesophagitis
H. Helicobacter pylori
I. Oesophageal varices
J. Adenocarcinoma
K. Duodenal ulceration
L. Campylobacter jejuni
A

B. Pernicious anaemia

How well did you know this?
1
Not at all
2
3
4
5
Perfectly
129
Q
Dilated veins in the lower oesophagus due to portal hypertension. These may rupture, leading to life threatening haematemesis. Bleeding may be stopped by a compression balloon, sclerotherapy, or applying elastic bands via an endoscope.
A. Barrett’s oesophagus
B. Pernicious anaemia
C. Squamous carcinoma
D. Gastric ulcer
E. Gastric cancer
F. Intestinal metaplasia
G. Reflux oesophagitis
H. Helicobacter pylori
I. Oesophageal varices
J. Adenocarcinoma
K. Duodenal ulceration
L. Campylobacter jejuni
A

I. Oesophageal varices

How well did you know this?
1
Not at all
2
3
4
5
Perfectly
130
Q
A genus of spiral flagellated Gram negative bacteria. Found in the stomach within the mucosa layer. It occurs in the majority of middle-aged people and causes progressive gastritis. Invariably present in duodenal ulceration and usually in gastric ulceration
A. Barrett’s oesophagus
B. Pernicious anaemia
C. Squamous carcinoma
D. Gastric ulcer
E. Gastric cancer
F. Intestinal metaplasia
G. Reflux oesophagitis
H. Helicobacter pylori
I. Oesophageal varices
J. Adenocarcinoma
K. Duodenal ulceration
L. Campylobacter jejuni
A

H. Helicobacter pylori

How well did you know this?
1
Not at all
2
3
4
5
Perfectly
131
Q
A breach in mucosa which extends through muscularis mucosa into submucosa or deeper
A. Peptic ulcer
B. Coeliac disease
C. Chronic gastritis
D. Intestinal metaplasia
E. Normal oesophagus
F. Pernicious anaemia
G. GORD
H. Barrett’s oesophagus
I. Normal stomach
J. Partial villus atrophy
K. H. pylori infection
A

A. Peptic ulcer

How well did you know this?
1
Not at all
2
3
4
5
Perfectly
132
Q
Present in almost all patients with duodenal ulcer and 70 % with gastric ulcer.
A. Peptic ulcer
B. Coeliac disease
C. Chronic gastritis
D. Intestinal metaplasia
E. Normal oesophagus
F. Pernicious anaemia
G. GORD
H. Barrett’s oesophagus
I. Normal stomach
J. Partial villus atrophy
K. H. pylori infection
A

K. H. pylori infection

How well did you know this?
1
Not at all
2
3
4
5
Perfectly
133
Q
Around 10 % eventually get primary lymphoma (less often, carcinoma) of the gut if not properly treated. HLA B8 is linked with this.
A. Peptic ulcer
B. Coeliac disease
C. Chronic gastritis
D. Intestinal metaplasia
E. Normal oesophagus
F. Pernicious anaemia
G. GORD
H. Barrett’s oesophagus
I. Normal stomach
J. Partial villus atrophy
K. H. pylori infection
A

B. Coeliac disease

How well did you know this?
1
Not at all
2
3
4
5
Perfectly
134
Q
The commonest cause of oesophagitis.
A. Peptic ulcer
B. Coeliac disease
C. Chronic gastritis
D. Intestinal metaplasia
E. Normal oesophagus
F. Pernicious anaemia
G. GORD
H. Barrett’s oesophagus
I. Normal stomach
J. Partial villus atrophy
K. H. pylori infection
A

G. GORD

How well did you know this?
1
Not at all
2
3
4
5
Perfectly
135
Q
Re-epithelialisation by metaplastic columnar epithelium with goblet cells
A. Peptic ulcer
B. Coeliac disease
C. Chronic gastritis
D. Intestinal metaplasia
E. Normal oesophagus
F. Pernicious anaemia
G. GORD
H. Barrett’s oesophagus
I. Normal stomach
J. Partial villus atrophy
K. H. pylori infection
A

H. Barrett’s oesophagus

How well did you know this?
1
Not at all
2
3
4
5
Perfectly
136
Q
A 40 year old male complaining of a long history of burning epigastric pain, worse on lying flat. Endoscopy and biopsy reveals inflamed squamous lining and increased basal cell proliferation
A. Oesophageal Adenocarcinoma
B. Duodenal Ulcer
C. Barretts Oesophagus
D. GORD
E. Oesophageal Varices
F. Active Chronic Gastritis
G. Acute Gastritis
H. Coeliac Disease
I. Pernicious Anaemia
J. Gastric Carcinoma
A

D. GORD

How well did you know this?
1
Not at all
2
3
4
5
Perfectly
137
Q
A 38 year old female with Rheumatoid Arthritis presents with a single episode of malaena. Investigations reveal erosions through out the stomach and a neutrophilic infiltrate in the superficial mucosa
A. Oesophageal Adenocarcinoma
B. Duodenal Ulcer
C. Barretts Oesophagus
D. GORD
E. Oesophageal Varices
F. Active Chronic Gastritis
G. Acute Gastritis
H. Coeliac Disease
I. Pernicious Anaemia
J. Gastric Carcinoma
A

G. Acute Gastritis

How well did you know this?
1
Not at all
2
3
4
5
Perfectly
138
Q
A 30 year old female complaining of diarhorrea and weight loss. Biopsy of duodenum shows increased intraepithelial cytotoxic T cells.
A. Oesophageal Adenocarcinoma
B. Duodenal Ulcer
C. Barretts Oesophagus
D. GORD
E. Oesophageal Varices
F. Active Chronic Gastritis
G. Acute Gastritis
H. Coeliac Disease
I. Pernicious Anaemia
J. Gastric Carcinoma
A

H. Coeliac Disease

How well did you know this?
1
Not at all
2
3
4
5
Perfectly
139
Q
A 60 year old male complaining of epigastric pain relieved by antacids and meals. He has a positive CLO test.	
A. Oesophageal Adenocarcinoma
B. Duodenal Ulcer
C. Barretts Oesophagus
D. GORD
E. Oesophageal Varices
F. Active Chronic Gastritis
G. Acute Gastritis
H. Coeliac Disease
I. Pernicious Anaemia
J. Gastric Carcinoma
A

B. Duodenal Ulcer

How well did you know this?
1
Not at all
2
3
4
5
Perfectly
140
Q
A 65 year old male with a long history of epigastric pain. Endoscopy reveals 3.2cm of columnar metaplasia in the lower oesophagus. Goblet cells are seen
A. Oesophageal Adenocarcinoma
B. Duodenal Ulcer
C. Barretts Oesophagus
D. GORD
E. Oesophageal Varices
F. Active Chronic Gastritis
G. Acute Gastritis
H. Coeliac Disease
I. Pernicious Anaemia
J. Gastric Carcinoma
A

C. Barretts Oesophagus

How well did you know this?
1
Not at all
2
3
4
5
Perfectly
141
Q
A 63 year old man presents with epigastric pain associated with dyspepsia. The pain gets worse at night and when he is hungry. He complains of nausea and flatulence. This patient is on NSAIDs.
A. Whipple's disease
B. Microsporidiosis
C. Tropical sprue
D. Barrett's oesophagus
E. Coeliac disease
F. Cryptosporidiosis
G. Lymphoma
H. Gastric ulcer
I. Duodenal ulcer
J. Helicobacter pylori
K. Pernicious anaemia
L. Gastro-oesophageal disease
M. Partial villous atrophy
N. Carcinoma of the oesophagus
O. Mucosal associated lymphoid tumour
P. Hiatus hernia
A

I. Duodenal ulcer

How well did you know this?
1
Not at all
2
3
4
5
Perfectly
142
Q
A 70 year old woman has progressive low retrosternal dysphagia, initially to solids and now also to liquids. She complains of chest pain and weight loss over the last 3 months. A social history reveals that she has been a heavy smoker for many years and drinks around 20 units of alcohol a week
A. Whipple's disease
B. Microsporidiosis
C. Tropical sprue
D. Barrett's oesophagus
E. Coeliac disease
F. Cryptosporidiosis
G. Lymphoma
H. Gastric ulcer
I. Duodenal ulcer
J. Helicobacter pylori
K. Pernicious anaemia
L. Gastro-oesophageal disease
M. Partial villous atrophy
N. Carcinoma of the oesophagus
O. Mucosal associated lymphoid tumour
P. Hiatus hernia
A

N. Carcinoma of the oesophagus

How well did you know this?
1
Not at all
2
3
4
5
Perfectly
143
Q
A 26 year old man presents with watery diarrhoea, abdominal cramps, nausea, vomiting and a low grade fever. It started 3 days after eating some undercooked meat at a barbecue.
A. Whipple's disease
B. Microsporidiosis
C. Tropical sprue
D. Barrett's oesophagus
E. Coeliac disease
F. Cryptosporidiosis
G. Lymphoma
H. Gastric ulcer
I. Duodenal ulcer
J. Helicobacter pylori
K. Pernicious anaemia
L. Gastro-oesophageal disease
M. Partial villous atrophy
N. Carcinoma of the oesophagus
O. Mucosal associated lymphoid tumour
P. Hiatus hernia
A

F. Cryptosporidiosis

How well did you know this?
1
Not at all
2
3
4
5
Perfectly
144
Q
A 66 year old man complaining of epigastric pain undergoes an endoscopy. The mucosa appears reddened in the antrum of the stomach. 13C is detected on a urea breath test.
A. Whipple's disease
B. Microsporidiosis
C. Tropical sprue
D. Barrett's oesophagus
E. Coeliac disease
F. Cryptosporidiosis
G. Lymphoma
H. Gastric ulcer
I. Duodenal ulcer
J. Helicobacter pylori
K. Pernicious anaemia
L. Gastro-oesophageal disease
M. Partial villous atrophy
N. Carcinoma of the oesophagus
O. Mucosal associated lymphoid tumour
P. Hiatus hernia
A

J. Helicobacter pylori

How well did you know this?
1
Not at all
2
3
4
5
Perfectly
145
Q
A 58 year old female presents with malnutrition. She complains of abdominal pain, weight loss and arthritis. She has steatorrhoea. A jejunal biopsy showed periodic acid-Schiff (PAS)-positive macrophages
A. Whipple's disease
B. Microsporidiosis
C. Tropical sprue
D. Barrett's oesophagus
E. Coeliac disease
F. Cryptosporidiosis
G. Lymphoma
H. Gastric ulcer
I. Duodenal ulcer
J. Helicobacter pylori
K. Pernicious anaemia
L. Gastro-oesophageal disease
M. Partial villous atrophy
N. Carcinoma of the oesophagus
O. Mucosal associated lymphoid tumour
P. Hiatus hernia
A

A. Whipple’s disease

How well did you know this?
1
Not at all
2
3
4
5
Perfectly
146
Q
A 35-year-old man presents with a long history of epigastric burning pain, made worse at night and when drinking hot liquids. Recently he has had difficulty swallowing solids. Endoscopy shows lower oesophageal erosions and strictures and pH demonstrates acidity.
A. Adenocarcinoma
B. Achalasia
C. Zollinger-Ellison syndrome
D. Bulbar palsy
E. Gastro-oesophageal reflux disease
F. Gastric ulcer
G. Mallory-Weiss tear
H. Barrett's oesophagus
I. Duodenal ulcer
J. Haemorrhagic gastritis
K. Gastroenteritis (Staphylococcus aureus)
L. Mucosal-associated lymphoid tumour
M. Gastroenteritis (Salmonella)
N. Pyloric stenosis
A

E. Gastro-oesophageal reflux disease

How well did you know this?
1
Not at all
2
3
4
5
Perfectly
147
Q
A 20-year-old student gives an 8 hour history of very frequent vomiting and epigastric cramping. O/E she is pale and shivering. Her serum WBC is normal.
A. Adenocarcinoma
B. Achalasia
C. Zollinger-Ellison syndrome
D. Bulbar palsy
E. Gastro-oesophageal reflux disease
F. Gastric ulcer
G. Mallory-Weiss tear
H. Barrett's oesophagus
I. Duodenal ulcer
J. Haemorrhagic gastritis
K. Gastroenteritis (Staphylococcus aureus)
L. Mucosal-associated lymphoid tumour
M. Gastroenteritis (Salmonella)
N. Pyloric stenosis
A

K. Gastroenteritis (Staphylococcus aureus)

How well did you know this?
1
Not at all
2
3
4
5
Perfectly
148
Q
A 30-year-old woman presents with haematemesis and diarrhoea. She has recurrent peptic ulceration and is taking omeprazole. Despite this, she has persistently high serum gastrin levels. Endoscopy shows a large 3cm actively bleeding ulcer in the duodenum.
A. Adenocarcinoma
B. Achalasia
C. Zollinger-Ellison syndrome
D. Bulbar palsy
E. Gastro-oesophageal reflux disease
F. Gastric ulcer
G. Mallory-Weiss tear
H. Barrett's oesophagus
I. Duodenal ulcer
J. Haemorrhagic gastritis
K. Gastroenteritis (Staphylococcus aureus)
L. Mucosal-associated lymphoid tumour
M. Gastroenteritis (Salmonella)
N. Pyloric stenosis
A

C. Zollinger-Ellison syndrome

How well did you know this?
1
Not at all
2
3
4
5
Perfectly
149
Q
A 50-year-old women presents with chest pain associated with regurgitation of solids and liquids equally, both occurring after swallowing. Diagnosis is confirmed by a characteristic ‘beak like’ tapering of the lower oesophagus on barium swallow and manometry shows failure of relaxation of the LOS.
A. Adenocarcinoma
B. Achalasia
C. Zollinger-Ellison syndrome
D. Bulbar palsy
E. Gastro-oesophageal reflux disease
F. Gastric ulcer
G. Mallory-Weiss tear
H. Barrett's oesophagus
I. Duodenal ulcer
J. Haemorrhagic gastritis
K. Gastroenteritis (Staphylococcus aureus)
L. Mucosal-associated lymphoid tumour
M. Gastroenteritis (Salmonella)
N. Pyloric stenosis
A

B. Achalasia

If you have achalasia, your LES (lower oesophageal sphincter) fails to open up during swallowing, which it’s supposed to do. This leads to a backup of food within your esophagus.

How well did you know this?
1
Not at all
2
3
4
5
Perfectly
150
Q
A 65-year-old woman presents with a 3 month history of anorexia, weight loss and epigastric pain. Blood tests reveal an iron deficiency anaemia. Endoscopy shows a thickened rigid gastric wall known as ‘leather bottle stomach’ indicating infiltration into all layers of the gastric wall. Numerous signet ring cells on biopsy diffusely infiltrate the mucosa.
A. Adenocarcinoma
B. Achalasia
C. Zollinger-Ellison syndrome
D. Bulbar palsy
E. Gastro-oesophageal reflux disease
F. Gastric ulcer
G. Mallory-Weiss tear
H. Barrett's oesophagus
I. Duodenal ulcer
J. Haemorrhagic gastritis
K. Gastroenteritis (Staphylococcus aureus)
L. Mucosal-associated lymphoid tumour
M. Gastroenteritis (Salmonella)
N. Pyloric stenosis
A

A. Adenocarcinoma

How well did you know this?
1
Not at all
2
3
4
5
Perfectly
151
Q
A 45 year old woman presents with large tongue and swelling of the legs. She has a high BP and urine dipstick reveals protein +++.The tissue from renal biopsy stains with Congo red dye and shows apple green birefringence under polarised light
A. Polyarteritis nodosa
B. Sarcoidosis
C. Scleroderma
D. Renal amyloidosis
E. Mixed connective tissue disease
F. Kawasaki's disease
G. Sjorgen's syndrome
H. Temporal arteritis
I. Systemic lupus erythematous
A

D. Renal amyloidosis

How well did you know this?
1
Not at all
2
3
4
5
Perfectly
152
Q
A 28 year old woman presents with malaise, weight loss, an erythematous rash on the face and joint pains. Both antinuclear antibodies (ANA) and double-stranded DNA (dsDNA) antibodies were found in the serum.
A. Polyarteritis nodosa
B. Sarcoidosis
C. Scleroderma
D. Renal amyloidosis
E. Mixed connective tissue disease
F. Kawasaki's disease
G. Sjorgen's syndrome
H. Temporal arteritis
I. Systemic lupus erythematous
A

I. Systemic lupus erythematous

153
Q
A 55 year old woman presents with severe, unremitting headache with scalp tenderness. Her ESR and CRP are raised. A biopsy reveals giant cells.
A. Polyarteritis nodosa
B. Sarcoidosis
C. Scleroderma
D. Renal amyloidosis
E. Mixed connective tissue disease
F. Kawasaki's disease
G. Sjorgen's syndrome
H. Temporal arteritis
I. Systemic lupus erythematous
A

H. Temporal arteritis

154
Q
A 40 year old man with previous hepatitis B infection presents with weight loss, muscle aches and abdominal pain. On examination he has high BP and urine dipstick reveals blood + and protein +
A. Polyarteritis nodosa
B. Sarcoidosis
C. Scleroderma
D. Renal amyloidosis
E. Mixed connective tissue disease
F. Kawasaki's disease
G. Sjorgen's syndrome
H. Temporal arteritis
I. Systemic lupus erythematous
A

A. Polyarteritis nodosa

A medium vessel vasculitis associated with Hep B.
It can affect any organic the body but common symptoms include weight loss, muscle aches, joint pains, skin rashes or nodules, abdominal pains, sometimes with blood in the stools, and numbness or tingling in the hands or feet. Kidney involvement with high blood pressure and haematuria is also often seen.

155
Q
A 30 year old Afrocaribbean woman presents with tender red nodules on the shins and legs. She also has joint pains in her feet and hands. Her blood test reveals a raised angiotensin converting enzyme (ACE) and Ca2+ level.
A. Polyarteritis nodosa
B. Sarcoidosis
C. Scleroderma
D. Renal amyloidosis
E. Mixed connective tissue disease
F. Kawasaki's disease
G. Sjorgen's syndrome
H. Temporal arteritis
I. Systemic lupus erythematous
A

B. Sarcoidosis

156
Q
A 35 year old female presents to her GP with unsightly, red, tender lesions on her shins. A subsequent CT scan shows evidence of enlarged glands in the lung hilar region, and nodular shadowing in the right middle lobe.
A. Sarcoidosis
B. Waldenström’s macroglobulinaemia
C. Myeloma associated amyloidosis
D. Bronchial carcinoma
E. Senile amyloidosis
F. Haemodialysis associated amyloidosis
G. Hogkin’s Lymphoma
H. Reactive amyloidosis
I. Hereditary amyloidosis
A

A. Sarcoidosis

157
Q
A 60 year old man, currently undergoing treatment for long-standing chronic renal failure, complaining of tingling in his wrist &amp; hand when he wakes in the morning
A. Sarcoidosis
B. Waldenström’s macroglobulinaemia
C. Myeloma associated amyloidosis
D. Bronchial carcinoma
E. Senile amyloidosis
F. Haemodialysis associated amyloidosis
G. Hogkin’s Lymphoma
H. Reactive amyloidosis
I. Hereditary amyloidosis
A

F. Haemodialysis associated amyloidosis

158
Q
A 70 year old woman is referred to hospital with signs of peripheral oedema and hepatosplenomegaly. Hospital investigations demonstrate a degree of bone erosion and high levels of circulating kappa uniform light chain
A. Sarcoidosis
B. Waldenström’s macroglobulinaemia
C. Myeloma associated amyloidosis
D. Bronchial carcinoma
E. Senile amyloidosis
F. Haemodialysis associated amyloidosis
G. Hogkin’s Lymphoma
H. Reactive amyloidosis
I. Hereditary amyloidosis
A

C. Myeloma associated amyloidosis

159
Q
A 32 year old man presents with a painless, enlarged axillary lymph node. Slight hepatosplenomegaly is noted on examination. Whilst the patient denies experiencing any night sweats, weight loss or fevers, bloods on admission show a raised ESR and abnormal liver biochemistry.
A. Sarcoidosis
B. Waldenström’s macroglobulinaemia
C. Myeloma associated amyloidosis
D. Bronchial carcinoma
E. Senile amyloidosis
F. Haemodialysis associated amyloidosis
G. Hogkin’s Lymphoma
H. Reactive amyloidosis
I. Hereditary amyloidosis
A

G. Hogkin’s Lymphoma

160
Q
A 64 year old woman with a history of chronic rheumatological disease presents to her GP complaining of abdominal discomfort – which is found to be due to hepatosplenomegaly. An ensuing liver biopsy stains positive with Congo Red stain.
A. Sarcoidosis
B. Waldenström’s macroglobulinaemia
C. Myeloma associated amyloidosis
D. Bronchial carcinoma
E. Senile amyloidosis
F. Haemodialysis associated amyloidosis
G. Hogkin’s Lymphoma
H. Reactive amyloidosis
I. Hereditary amyloidosis
A

H. Reactive amyloidosis

161
Q

List 5 causes of acute pancreatitis?

A

Gallstones, Alcohol, Mumps, steroids, trauma

Autoimmune, scorpian sting, hypercalcaemia, ERCP, drugs (thiazides, azathioprine)

162
Q
Which of the following is not a complication of acute pancreatitis?
A. Psudocyst
B. Shock
C. Abscess
D. Hypocalaemia
E. Pancreatic ulcer
A

E. Pancreatic ulcer

163
Q

What is a pseudocyst?

A

A collection of fluid rich in pancreatic enzymes or necrotic material, without an epithelial lining. It can: resolve, compress other structures, become infected or perforate

164
Q
Which of the following is not a complication of chronic pancreatitis?
A. Diabetes
B. Psudocyst
C. Carcinoma of the pancreas
D. Metabolic alkalosis 
E. Malabsorption
A

D. Metabolic alkalosis

165
Q
Large number of IgG4 positive plasma cells is seen in what disease?
A. Chronic pancreatitis
B. Lymphoma 
C. Autoimmune pancreatitis
D. Cholecystitis 
E. Pancreatic ductal carcinoma
A

C. Autoimmune pancreatitis

166
Q
What is the most common type of malignant tumour of the pancreas? 
A. Ductal carcinoma 
B. Acinar carcinoma 
C. Serious cystadenoma 
D. Mucinous cystic neoplasm
A

A. Ductal carcinoma

Risk factors: Smoking, BMI, dietary factors, chronic pancreatitis, diabetes

On microscopy: adenocarcinoma = mucin secreting glands set in desmoplastic stroma.

Most commonly in head of pancreas

167
Q

What mutation is associated with 95% of pancreatic ductal carcinomas?

A

K-ras mutation

168
Q

Name the 7 major risk factors associated with atherosclerosis? And which is the most significant independent risk factor?

A

Age, gender, genetics, hyperlipidaemia, hypertension, smoking, DM

Genetics is the most significant independent risk factor

169
Q

What are the three consequences of atheroma?

A

Rupture, erosion, haemorrhage into plaque.

170
Q

Which of the following is not a feature of a vulnerable plaque?
A. Lots of foam cells
B. Few smooth muscle cells
C. Thick fibrous cap

A

C. Thick fibrous cap

Thin fibrous cap make a plaque more vulnerable to rupture

171
Q

Which of the following is not true regarding restrictive cardiomyopathy?
A. Can be caused by sarcoidosis
B. There is thickening of the septum
C. There is impaired ventricular compliance
D. There is a normal heart size but big atria

A

B. There is thickening of the septum

This is a feature of hypertrophic cardiomyopathy

172
Q
A 63 year old lady presents to A and E with acute abdominal pain. She has been vomiting. On examination her abdomen is distended and you hear tinkling bowel sounds. Abdominal X-ray shows an inverted U loop of bowel.
A. Diverticular disease
B. Sigmoid volvulus
C. Diverticulitis
D. Pseudomembranous colitis
E. Ischaemic colitis
F. Tubular adenoma
G. Gastroenteritis
H. Angiodysplasia
I. Colorectal carcinoma
J. Ulcerative colitis
K. Crohn's disease
L. Intestinal TB
A

B. Sigmoid volvulus

173
Q
A 70 year old gentlemen presents to his GP with left sided colicky abdominal pain which is relieved by defecation. He admits to passing constipated stools.
A. Diverticular disease
B. Sigmoid volvulus
C. Diverticulitis
D. Pseudomembranous colitis
E. Ischaemic colitis
F. Tubular adenoma
G. Gastroenteritis
H. Angiodysplasia
I. Colorectal carcinoma
J. Ulcerative colitis
K. Crohn's disease
L. Intestinal TB
A

A. Diverticular disease

174
Q
A 65 year old lady presents to A and E with left sided abdominal pain which is relieved by defecation. On examination her abdomen is tender over the left iliac fossa and she is pyrexial. Her ESR and WCC are both raised.
A. Diverticular disease
B. Sigmoid volvulus
C. Diverticulitis
D. Pseudomembranous colitis
E. Ischaemic colitis
F. Tubular adenoma
G. Gastroenteritis
H. Angiodysplasia
I. Colorectal carcinoma
J. Ulcerative colitis
K. Crohn's disease
L. Intestinal TB
A

C. Diverticulitis

175
Q
A 76 year old lady was admitted to hospital with community acquired pneumonia. She was treated with Cefuroxime and Clarythromycin. 5 days later she developed bloody diarrhoea and her fever returned.
A. Diverticular disease
B. Sigmoid volvulus
C. Diverticulitis
D. Pseudomembranous colitis
E. Ischaemic colitis
F. Tubular adenoma
G. Gastroenteritis
H. Angiodysplasia
I. Colorectal carcinoma
J. Ulcerative colitis
K. Crohn's disease
L. Intestinal TB
A

D. Pseudomembranous colitis

176
Q
A 26 year old gentleman presents to A and E with acute abdominal pain. He has lost 5 kg of weight in the last 2 weeks, and has been passing bloody diarrhoea with mucus. He is pyrexial and on examination is noted to have angular stomatitis.
A. Diverticular disease
B. Sigmoid volvulus
C. Diverticulitis
D. Pseudomembranous colitis
E. Ischaemic colitis
F. Tubular adenoma
G. Gastroenteritis
H. Angiodysplasia
I. Colorectal carcinoma
J. Ulcerative colitis
K. Crohn's disease
L. Intestinal TB
A

K. Crohn’s disease

177
Q
A 60-year-old patient presents with chronic left-sided abdominal discomfort and the passage of fresh blood from the rectum. He denies weight loss and on examination has no palpable masses. However a colonoscopy reveals pouches of mucosa extruding through the muscle layer.
A. Coeliac disease
B. Colon carcinoma
C. Hyperplastic Polyp
D. Acute appendicitis
E. Hirschprung’s Disease
F. Crohn’s Disease
G. Ulcerative colitis
H. Diverticular Disease
I. Pseudomembranous colitis
A

H. Diverticular Disease

178
Q
A 70-year-old man who has been receiving broad-spectrum antibiotic treatment develops profuse diarrhoea and frequent episodes of incontinence. A biopsy of the colon reveals a membrane like material covering the surface composed of mucin, fibrin, polymorphs and what seems to be debris of leukocytes and mucosal epithelial cells.
A. Coeliac disease
B. Colon carcinoma
C. Hyperplastic Polyp
D. Acute appendicitis
E. Hirschprung’s Disease
F. Crohn’s Disease
G. Ulcerative colitis
H. Diverticular Disease
I. Pseudomembranous colitis
A

I. Pseudomembranous colitis

179
Q
A 68-year-old man presents with rectal bleeding of bright red blood and constipation. He previously has been diagnosed with polyps in his colon and recently has lost a lot of weight. An abdominal examination reveals a mass in the right lumbar region which is non-tender
A. Coeliac disease
B. Colon carcinoma
C. Hyperplastic Polyp
D. Acute appendicitis
E. Hirschprung’s Disease
F. Crohn’s Disease
G. Ulcerative colitis
H. Diverticular Disease
I. Pseudomembranous colitis
A

B. Colon carcinoma

180
Q
A 25-year-old woman complains of right iliac fossa pain, diarrhoea and weight loss worsening over several weeks. Laparotomy reveals an oedematous, reddened terminal ileum and a biopsy uncovers transmural inflammation with the presence of granulomas
A. Coeliac disease
B. Colon carcinoma
C. Hyperplastic Polyp
D. Acute appendicitis
E. Hirschprung’s Disease
F. Crohn’s Disease
G. Ulcerative colitis
H. Diverticular Disease
I. Pseudomembranous colitis
A

F. Crohn’s Disease

181
Q
A 35-year-old woman presents with diarrhoea and lower abdominal pain. Examination of her stools reveals blood and mucus. A biopsy reveals a continuous superficial ulceration of the colon
A. Coeliac disease
B. Colon carcinoma
C. Hyperplastic Polyp
D. Acute appendicitis
E. Hirschprung’s Disease
F. Crohn’s Disease
G. Ulcerative colitis
H. Diverticular Disease
I. Pseudomembranous colitis
A

G. Ulcerative colitis

182
Q
A 77 year old man was referred by his G.P. after reporting the passing of large volumes of mucus. Endoscopy revealed a 6cm sessile cauliflower-like mass covered by dysplastic columnar epithelium in the rectum. His serum potassium is 3.3 mmol/l.
A. Perianal haematoma
B. Anterior rectocoele
C. Villous adenoma
D. Adenocarcinoma Duke's stage C1
E. Ischaemic colitis
F. Adenocarcinoma Duke's stage B
G. Haemorrhoids
H. Diverticulitis
I. Ulcerative colitis
J. Diverticular disease
K. Benign adenoma
L. Irritable bowel syndrome
M. Angiodysplasia
N. Anorectal abscess
O. Crohn's disease
P. Appendicitis
A

C. Villous adenoma

183
Q
A 62 year old housewife returns to your outpatient clinic following another incidence of the passing of blood. Previous sigmoidoscopy, DRE and barium enema’s have failed to identify any lesion and she denies weight loss and diarrhoea. However, blood tests show a microcytic anaemia.
A. Perianal haematoma
B. Anterior rectocoele
C. Villous adenoma
D. Adenocarcinoma Duke's stage C1
E. Ischaemic colitis
F. Adenocarcinoma Duke's stage B
G. Haemorrhoids
H. Diverticulitis
I. Ulcerative colitis
J. Diverticular disease
K. Benign adenoma
L. Irritable bowel syndrome
M. Angiodysplasia
N. Anorectal abscess
O. Crohn's disease
P. Appendicitis
A

M. Angiodysplasia

184
Q
An 84 year old gentleman presents to A&amp;E after suffering acute cramp-like abdominal pain lasting several hours which was followed shortly by rectal bleeding. Colonoscopy showed an oedematous thickening of the bowel wall with areas of necrotic ulceration which was confined to the mucosal layers. On examination, the gentleman was found to have an irregularly irregular heart beat.
A. Perianal haematoma
B. Anterior rectocoele
C. Villous adenoma
D. Adenocarcinoma Duke's stage C1
E. Ischaemic colitis
F. Adenocarcinoma Duke's stage B
G. Haemorrhoids
H. Diverticulitis
I. Ulcerative colitis
J. Diverticular disease
K. Benign adenoma
L. Irritable bowel syndrome
M. Angiodysplasia
N. Anorectal abscess
O. Crohn's disease
P. Appendicitis
A

E. Ischaemic colitis

185
Q
A 34 year old female presents with diarrhoea, abdominal pain and weight loss. Colonoscopy reveals inflammation of the mucosa which has a cobblestoned appearance. Biopsy showed transmural involvement and the presence of non-caseating granulomas.	
A. Perianal haematoma
B. Anterior rectocoele
C. Villous adenoma
D. Adenocarcinoma Duke's stage C1
E. Ischaemic colitis
F. Adenocarcinoma Duke's stage B
G. Haemorrhoids
H. Diverticulitis
I. Ulcerative colitis
J. Diverticular disease
K. Benign adenoma
L. Irritable bowel syndrome
M. Angiodysplasia
N. Anorectal abscess
O. Crohn's disease
P. Appendicitis
A

O. Crohn’s disease

186
Q
Your consultant is urgently called away from out-patients and asks you to break bad news to a patient. He is distracted and has little time to inform you about the case but mentions that the original presentation was for rectal bleeding and the patient has a 40% chance of surviving 5 years.
A. Perianal haematoma
B. Anterior rectocoele
C. Villous adenoma
D. Adenocarcinoma Duke's stage C1
E. Ischaemic colitis
F. Adenocarcinoma Duke's stage B
G. Haemorrhoids
H. Diverticulitis
I. Ulcerative colitis
J. Diverticular disease
K. Benign adenoma
L. Irritable bowel syndrome
M. Angiodysplasia
N. Anorectal abscess
O. Crohn's disease
P. Appendicitis
A

D. Adenocarcinoma Duke’s stage C1

187
Q
A 65 year old heavy smoker. He has been progressively short of breath over a few years. He has a smooth liver edge 2 cms below the costal margin.
A. Chronic myeloid leukaemia
B. Congestive heart failure
C. Hameochromatosis
D. Cirrhosis with hepatoma
E. Gaucher's disease
F. Chronic lymphocytic leukaemia
G. Portal vein thrombosis
H. Severe emphysema
I. Sarcoidosis
J. Liver abscess
K. Polycythaemia rubra vera
A

H. Severe emphysema

188
Q
A 60 year old woman is found to have hepatomegaly. She has a history of moderate alcohol use. She had an anterior MI 2 years ago. On examination she has ankle oedema, elevated JVP and 3 spider naevi on her chest.
A. Chronic myeloid leukaemia
B. Congestive heart failure
C. Hameochromatosis
D. Cirrhosis with hepatoma
E. Gaucher's disease
F. Chronic lymphocytic leukaemia
G. Portal vein thrombosis
H. Severe emphysema
I. Sarcoidosis
J. Liver abscess
K. Polycythaemia rubra vera
A

B. Congestive heart failure

189
Q
A 78 year old woman attends complaining of widespread itching. Examination reveals hepatosplenomegaly. The patient appears plethoric with no lymphadenopathy.
A. Chronic myeloid leukaemia
B. Congestive heart failure
C. Hameochromatosis
D. Cirrhosis with hepatoma
E. Gaucher's disease
F. Chronic lymphocytic leukaemia
G. Portal vein thrombosis
H. Severe emphysema
I. Sarcoidosis
J. Liver abscess
K. Polycythaemia rubra vera
A

K. Polycythaemia rubra vera

190
Q
A 50 year old with haemophilia and hepatitis C presents with weight loss and abdominal discomfort. He is mildly icteric with features of chronic liver disease and a large left lobe of the liver.
A. Chronic myeloid leukaemia
B. Congestive heart failure
C. Hameochromatosis
D. Cirrhosis with hepatoma
E. Gaucher's disease
F. Chronic lymphocytic leukaemia
G. Portal vein thrombosis
H. Severe emphysema
I. Sarcoidosis
J. Liver abscess
K. Polycythaemia rubra vera
A

D. Cirrhosis with hepatoma

191
Q
A 73 year old woman attends complaining of recent onset of tiredness. She is pale and has hepatosplenomegaly and generalised lymphadenopathy in the neck, axillae and groins.
A. Chronic myeloid leukaemia
B. Congestive heart failure
C. Hameochromatosis
D. Cirrhosis with hepatoma
E. Gaucher's disease
F. Chronic lymphocytic leukaemia
G. Portal vein thrombosis
H. Severe emphysema
I. Sarcoidosis
J. Liver abscess
K. Polycythaemia rubra vera
A

F. Chronic lymphocytic leukaemia

192
Q
A 25 year old student presents to A&amp;E complaining of abdominal pain, diarrhoea, nausea and vomiting over the past few days. On examination he appears jaundiced and has dark urine and pale stools. Blood tests reveal raised levels of AST, ALT and bilirubin and a raised ESR. Further questioning reveals that he has returned from South East Asia three weeks previously.
A. Alcoholic liver disease
B. Autoimmune hepatitis
C. Hepatitis A
D. Wilson's disease
E. Hepatitis B
F. Primary biliary cirrhosis
G. Hereditary haemochromatosis
H. Hepatocellular carcinoma
A

C. Hepatitis A

193
Q
A 45 year old lady complains of pruritis. Investigations reveal the presence of antimitochondrial antibodies and elevated levels of alkaline phosphatase.
A. Alcoholic liver disease
B. Autoimmune hepatitis
C. Hepatitis A
D. Wilson's disease
E. Hepatitis B
F. Primary biliary cirrhosis
G. Hereditary haemochromatosis
H. Hepatocellular carcinoma
A

F. Primary biliary cirrhosis

194
Q
A 17 year old female presents to outpatients complaining of increasing confusion and a mild tremor. On examination you notice signs of liver disease. Investigations reveal a haemolytic anaemia. Further examination reveals a green-brown discolouration at the peripheries of her corneas.
A. Alcoholic liver disease
B. Autoimmune hepatitis
C. Hepatitis A
D. Wilson's disease
E. Hepatitis B
F. Primary biliary cirrhosis
G. Hereditary haemochromatosis
H. Hepatocellular carcinoma
A

D. Wilson’s disease

195
Q
A 45 year old male was admitted to hospital after having a fall. Investigations reveal a leucocytosis with elevated bilirubin and transferases. Levels were albumin, folate and vitamin B12 were low. Clotting studies showed a prolonged prothrombin time.
A. Alcoholic liver disease
B. Autoimmune hepatitis
C. Hepatitis A
D. Wilson's disease
E. Hepatitis B
F. Primary biliary cirrhosis
G. Hereditary haemochromatosis
H. Hepatocellular carcinoma
A

A. Alcoholic liver disease

196
Q
Routine biochemistry for a 32 year old man revealed elevated levels of serum iron and ferratin but reduced levels of total iron binding capacity. Other serum liver biochemistry investigations were normal. Further questioning revealed that his uncle had a ‘blood’ disorder.
A. Alcoholic liver disease
B. Autoimmune hepatitis
C. Hepatitis A
D. Wilson's disease
E. Hepatitis B
F. Primary biliary cirrhosis
G. Hereditary haemochromatosis
H. Hepatocellular carcinoma
A

G. Hereditary haemochromatosis

197
Q
A 30 year old female patient presents with abdominal pain, nausea and vomiting. Upon examination the patient has a tender hepatomegaly and ascites (with a high protein content) Abdominal CT demonstrated hepatic vein occlusion with diffuse abnormal parenchyma on contrast enhancement
A. Alpha1 antitrypsin deficiency
B. Hepatitis B
C. Primary sclerosing cholangitis
D. Ascending cholangitis
E. Hepatocellular carcinoma
F. Hereditary haemochromatosis
G. Wilson's disease
H. Primary biliary cirrhosis
I. Hepatitis C
J. Budd-Chiari syndrome
K. Alcoholic hepatitis
A

J. Budd-Chiari syndrome

198
Q
A 35 year old patient with known Hepatitis B visits the doctor complaining of recent weight loss, loss of appetite, fevers and an ache in the right hypochondrium. On examination an enlarged, irregular, tender liver can be palpated.
A. Alpha1 antitrypsin deficiency
B. Hepatitis B
C. Primary sclerosing cholangitis
D. Ascending cholangitis
E. Hepatocellular carcinoma
F. Hereditary haemochromatosis
G. Wilson's disease
H. Primary biliary cirrhosis
I. Hepatitis C
J. Budd-Chiari syndrome
K. Alcoholic hepatitis
A

E. Hepatocellular carcinoma

199
Q
A 15 year old boy presents with signs and symptoms of chronic liver disease. Upon examination Kayser-Fleisher rings are seen in the patients corneas.	
A. Alpha1 antitrypsin deficiency
B. Hepatitis B
C. Primary sclerosing cholangitis
D. Ascending cholangitis
E. Hepatocellular carcinoma
F. Hereditary haemochromatosis
G. Wilson's disease
H. Primary biliary cirrhosis
I. Hepatitis C
J. Budd-Chiari syndrome
K. Alcoholic hepatitis
A

G. Wilson’s disease

200
Q
A 22 year old patient presents to his doctor complaining of week long flu like symptoms, nausea, anorexia and fatigue. The patient appears jaundiced upon examination with mild enlargement and slight tenderness of the liver. Lab results demonstrate a raised AST (512 IU/L) and ALT (425IU/L) and an acute phase IgM response has been demonstrated. The patient recalls visiting a dodgy tattoo parlour 3 weeks ago to engrave the name of his mum on his arm.
A. Alpha1 antitrypsin deficiency
B. Hepatitis B
C. Primary sclerosing cholangitis
D. Ascending cholangitis
E. Hepatocellular carcinoma
F. Hereditary haemochromatosis
G. Wilson's disease
H. Primary biliary cirrhosis
I. Hepatitis C
J. Budd-Chiari syndrome
K. Alcoholic hepatitis
A

B. Hepatitis B

201
Q
A 42 year old woman presents to the doctor complaining of severe itching and recent onset of jaundice. The patient had been experiencing milder pruritis and bouts of fatigue for the last year. Hepatomegaly and pigmented xantholasma are demonstrated upon examination. Serum IgM is high and apart from a high serum Alk Phosphate, other liver biochemistry is normal. Upon ELISA screen, mitochondrial antibodies are seen. (in titre of 1:200)
A. Alpha1 antitrypsin deficiency
B. Hepatitis B
C. Primary sclerosing cholangitis
D. Ascending cholangitis
E. Hepatocellular carcinoma
F. Hereditary haemochromatosis
G. Wilson's disease
H. Primary biliary cirrhosis
I. Hepatitis C
J. Budd-Chiari syndrome
K. Alcoholic hepatitis
A

H. Primary biliary cirrhosis

202
Q
A 25 year old diabetic woman presents with a week long history of an itch in her groin accompanied by a ‘cheese like’ discharge. The vagina and vulva were red and sore, and strings of mycelium were shown on microscopy.
A. Candidiasis
B. Polycystic ovary syndrome
C. Endometriosis
D. Cervical polyps
E. Herpes virus infection
F. Endometritis
G. Salpingitis
H. Arias-Stella phenomenon
I. Fibroids
J. Ectopic pregnancy
A

A. Candidiasis

203
Q
A 30 year old woman presents with lower abdominal pain accompanied by fever. She has been using an intra uterine contraceptive device since her wedding three years ago.
A. Candidiasis
B. Polycystic ovary syndrome
C. Endometriosis
D. Cervical polyps
E. Herpes virus infection
F. Endometritis
G. Salpingitis
H. Arias-Stella phenomenon
I. Fibroids
J. Ectopic pregnancy
A

F. Endometritis

204
Q
A 40 year old woman has always known cramping pain associated with her periods – which have usually been heavy. Recently this pain has become constant throughout the month, and her periods have become more frequent. She claims never to have used oral contraception and has no children. She is abstaining from sexual intercourse as it is too painful.
A. Candidiasis
B. Polycystic ovary syndrome
C. Endometriosis
D. Cervical polyps
E. Herpes virus infection
F. Endometritis
G. Salpingitis
H. Arias-Stella phenomenon
I. Fibroids
J. Ectopic pregnancy
A

C. Endometriosis

205
Q
A 23 year old presents with worsening vaginal bleeding and abdominal pain. This started about a month ago and she has not had sex since the bleeding started. She has missed her last two periods. Passing water and defecation are painful, and she has noticed some pain in her shoulders.
A. Candidiasis
B. Polycystic ovary syndrome
C. Endometriosis
D. Cervical polyps
E. Herpes virus infection
F. Endometritis
G. Salpingitis
H. Arias-Stella phenomenon
I. Fibroids
J. Ectopic pregnancy
A

J. Ectopic pregnancy

206
Q
A 30 year old woman attends a fertility clinic as her and her husband are unable to conceive. For the last few years her periods have been fairly heavy and prolonged. She has stopped wearing skirts because she thinks her ankles look fat.	
A. Candidiasis
B. Polycystic ovary syndrome
C. Endometriosis
D. Cervical polyps
E. Herpes virus infection
F. Endometritis
G. Salpingitis
H. Arias-Stella phenomenon
I. Fibroids
J. Ectopic pregnancy
A

I. Fibroids

207
Q
What is the most common malignant tumour of the liver?
A. Hepatocellular carcinoma 
B. Hepatoblastima 
C. Cholangiocarcinoma 
D. Secondary tumours 
E. Haemangiosarcoma
A

D. Secondary tumours

208
Q
What is the commonest neoplasm of the oesophagus in the world?
A. Adenocarcinoma 
B. Barret's oesphagus 
C. Squamous cell carcinoma 
D. MALT
A

C. Squamous cell carcinoma
This is the most common worldwide. It is associated with smoking and alcohol. Histology will show keratin and intercellular bridges

Adenocarcinoma is most common in the UK

209
Q

What is the definition of a gastric ulcer?

A

Loss of surface epithelium; the depth of loss goes beyond the muscularis mucosa

210
Q

What is the definition of Dukes Stage B?

A

B = through the wall of the bowel

A = confined to bowel wall 
B = through the wall of the bowel 
C = lymph node involvement 
D = Distant metastasis
211
Q

Is Familial Adenomatous Polyposis dominant or recessive?

A

Autosomal dominant

1000s of polyps with very high likelihood of becoming cancer within 10-15 years.

212
Q
Which of the following is not a feature of Gardner's syndrome?
A. Multiple polpys
B. Epidermoid cysts
C. Multiple osteomas of skull 
D. Shallow ulcers 
E. Desmoid tumours
A

D. Shallow ulcers

Gardner’s syndrome has the same clinical pathological features as FAP with a high cancer risk. However there are some distinctive extra-intestinal manifestations to remember!

213
Q

Abnormal cells are seen in the superficial 1/3rd of the cervical epithelium. What CIN grade is this?

A

CIN 1= mild dyskaryosis

CIN2 = 2/3rds epithelium
CIN 3 = >2/3rds epithelium
Carcinoma in situ = full thickness; no penetration to surrounding tissue. Going through the BM makes in carcinoma in situ rather than CIN

214
Q
In cervical histology, if there is invasion through the basement membrane what stage it it?
A. CIN1
B. CIN2
C. CIN3
D. CIN4
E. Carcinoma in situ
A

E. Carcinoma in situ

215
Q

How does HPV transform cells?

A

-E6 and E7 are proteins encoded by the virus and they bind to tumour suppressor genes, p53 and retinoblastoma gene. By doing this it interferes with apoptosis

216
Q

What is the most common type of ovarian tumour?

A

Serous

217
Q
Differentiation of this is regulated via the RANK gene product
A. Diaphysis
B. Woven
C. Epiphysis
D. Periosteum
E. Osteoblast
F. Lamellar
G. Osteocyte
H. Cortical
I. Cancellous
J. Metaphysis
K. Osteoclast
L. Endosteum
A

K. Osteoclast

218
Q
Covers cortical surface of bone and delivers blood supply.
A. Diaphysis
B. Woven
C. Epiphysis
D. Periosteum
E. Osteoblast
F. Lamellar
G. Osteocyte
H. Cortical
I. Cancellous
J. Metaphysis
K. Osteoclast
L. Endosteum
A

D. Periosteum

219
Q
Bone-forming cells which line endosteal surfaces.
A. Diaphysis
B. Woven
C. Epiphysis
D. Periosteum
E. Osteoblast
F. Lamellar
G. Osteocyte
H. Cortical
I. Cancellous
J. Metaphysis
K. Osteoclast
L. Endosteum
A

E. Osteoblast

220
Q
This type of bone is 80-90% calcified and its function is mainly mechanical and protective.
A. Diaphysis
B. Woven
C. Epiphysis
D. Periosteum
E. Osteoblast
F. Lamellar
G. Osteocyte
H. Cortical
I. Cancellous
J. Metaphysis
K. Osteoclast
L. Endosteum
A

H. Cortical

221
Q
This type of bone is immature and usually pathological.
A. Diaphysis
B. Woven
C. Epiphysis
D. Periosteum
E. Osteoblast
F. Lamellar
G. Osteocyte
H. Cortical
I. Cancellous
J. Metaphysis
K. Osteoclast
L. Endosteum
A

B. Woven

222
Q
A 45-year-old Asian female presents with generalised bone pain. Histological findings are of increased osteoid volume with widened osteoid seams and reduced mineralization. Her diet is vegan.
A. Osteomalacia
B. Osteogenesis imperfecta
C. Primary osteosarcoma
D. Osteoporosis
E. Osteopetrosis
F. Chondrosarcoma
G. Osteoarthritis
H. Ewing's sarcoma
I. Osteomyelitis
A

A. Osteomalacia

223
Q
A 15-year-old male presents with a 2-month history of increasing pain in his right upper arm. Bone biopsy reveals sheets of cells with small, primitive nuclei and scanty cytoplasm. A positive immunoreactivity is seen with the MIC2 (CD99) antibody.
A. Osteomalacia
B. Osteogenesis imperfecta
C. Primary osteosarcoma
D. Osteoporosis
E. Osteopetrosis
F. Chondrosarcoma
G. Osteoarthritis
H. Ewing's sarcoma
I. Osteomyelitis
A

H. Ewing’s sarcoma

224
Q
Electronmicroscopy shows bone thinning and wide separation of trabecular plates in a postmenopausal Caucasian female of slender build.
A. Osteomalacia
B. Osteogenesis imperfecta
C. Primary osteosarcoma
D. Osteoporosis
E. Osteopetrosis
F. Chondrosarcoma
G. Osteoarthritis
H. Ewing's sarcoma
I. Osteomyelitis
A

D. Osteoporosis

225
Q
Mottled rarefaction and lifting of the periosteum are seen on X-ray of the femur. Bone biopsy reveals an intense neutrophilic inflammatory infiltrate.
A. Osteomalacia
B. Osteogenesis imperfecta
C. Primary osteosarcoma
D. Osteoporosis
E. Osteopetrosis
F. Chondrosarcoma
G. Osteoarthritis
H. Ewing's sarcoma
I. Osteomyelitis
A

I. Osteomyelitis

226
Q
A 20-year-old male complains of a progressively enlarging painful mass on his right upper arm. Radiology demonstrates a lytic lesion of the proximal humerus with an accompanying Codman triangle. Microscopically, pleomorphic mesenchymal cells producing dark-staining osteoid are seen.
A. Osteomalacia
B. Osteogenesis imperfecta
C. Primary osteosarcoma
D. Osteoporosis
E. Osteopetrosis
F. Chondrosarcoma
G. Osteoarthritis
H. Ewing's sarcoma
I. Osteomyelitis
A

C. Primary osteosarcoma

227
Q
A 33 year-old women from Somalia, who traditionally wears a veil, presents to her GP with a waddling gait and weakness of the proximal muscles. She has clinical features of tetany and a low serum 25-hydroxyvitamin D3. On skeletal X-ray linear areas of low density are seen surrounded by sclerotic borders
A. Secondary hyperparathyroidism with chronic renal osteodystrophy
B. Osteitis fibrosa
C. Osteoporosis
D. Familial hypocalcuric hypercalcaemia
E. Tertiary hyperparathyroidism
F. Cushing's syndrome
G. Paget's disease
H. Primary hypthyroidism (myxoedema)
I. Cushing's disease
J. Osteomalacia
K. Primary hyperparathyroidism
A

J. Osteomalacia

228
Q
88 year-old lady presents to A&amp;E after slipping on Ice outside her door on the way to the post office. X-ray shows a fractured left femoral head and her bone density appears to be low
A. Secondary hyperparathyroidism with chronic renal osteodystrophy
B. Osteitis fibrosa
C. Osteoporosis
D. Familial hypocalcuric hypercalcaemia
E. Tertiary hyperparathyroidism
F. Cushing's syndrome
G. Paget's disease
H. Primary hypthyroidism (myxoedema)
I. Cushing's disease
J. Osteomalacia
K. Primary hyperparathyroidism
A

C. Osteoporosis

229
Q
A 92 year-old man has asymptomatic elevation of alkaline phosphatase. Skull X-ray reveals osteoporosis circumscripta. On examination he has bowed tibia and skull changes. He also complains of a recent onset of deafness
A. Secondary hyperparathyroidism with chronic renal osteodystrophy
B. Osteitis fibrosa
C. Osteoporosis
D. Familial hypocalcuric hypercalcaemia
E. Tertiary hyperparathyroidism
F. Cushing's syndrome
G. Paget's disease
H. Primary hypthyroidism (myxoedema)
I. Cushing's disease
J. Osteomalacia
K. Primary hyperparathyroidism
A

G. Paget’s disease

230
Q
A 24 year old police woman attends the clinic as her GP suspects she may have a parathyroid tumour. She has raised PTH and serum calcium. After a 24hr urinary collection it is noted the patient has a low urine calcium output
A. Secondary hyperparathyroidism with chronic renal osteodystrophy
B. Osteitis fibrosa
C. Osteoporosis
D. Familial hypocalcuric hypercalcaemia
E. Tertiary hyperparathyroidism
F. Cushing's syndrome
G. Paget's disease
H. Primary hypthyroidism (myxoedema)
I. Cushing's disease
J. Osteomalacia
K. Primary hyperparathyroidism
A

D. Familial hypocalcuric hypercalcaemia

Dominantly inherited condition caused by a mutation of the calcium sensing receptor. The kidney resorbs calcium very avidly = hypocalciuria and hypercalcaemia

231
Q
You see a new patient for the first time at the surgery, a 14 year old boy, complaining of pain in his right leg for 9 months and contracture of the right knee which has developed over this period. The child looks stunted and you can see bowing of the lower extremities in ambulators. His notes have not arrived yet from his previous doctor but his mother tells you ‘he was born 3 weeks premature with two small kidneys and three failed transplants means he has dialysis four times a day’. His blood results later show a raised PTH and phostate, low calcium and 1,25(OH)vitamin D. He is also acidotic.
A. Secondary hyperparathyroidism with chronic renal osteodystrophy
B. Osteitis fibrosa
C. Osteoporosis
D. Familial hypocalcuric hypercalcaemia
E. Tertiary hyperparathyroidism
F. Cushing's syndrome
G. Paget's disease
H. Primary hypthyroidism (myxoedema)
I. Cushing's disease
J. Osteomalacia
K. Primary hyperparathyroidism
A

A. Secondary hyperparathyroidism with chronic renal osteodystrophy

Renal osteodystrophy is a bone disease that occurs when your kidneys fail to maintain proper calcium and phosphorous in the blood.
The kidneys stop making vitamin D so the body cannot absorb calcium.

232
Q
An 18 year old student presents to his GP with focal pain in his left fore-arm which is tender to touch and worsens at night. The pain is relieved with aspirin. An X-ray shows a 1cm are of radio-lucency in the tibia surrounded by dense bone.
A. Osteochondroma
B. Osteosarcoma
C. Fibrous dysplasia
D. Metastases
E. Osteitis
F. Osteoarthritis
G. Simple bone cyst
H. Trauma
I. Rheumatoid arthritis
J. Osteoid osteoma
K. Ewing's tumour
L. Osteoporosis
M. Chondrosarcoma
N. Echondroma
O. Osteoclastoma
A

J. Osteoid osteoma

Benign tumour that arises from osteoblasts. Often a dull pain that gets worse at night. The pain can be relieved by over the counter painkillers

233
Q
A 14 year old boy complains to you of a painless lump on his left thigh, just above the knee which is slowly growing. His past medical history reveals that he fractured his femur in the same location several years before.
A. Osteochondroma
B. Osteosarcoma
C. Fibrous dysplasia
D. Metastases
E. Osteitis
F. Osteoarthritis
G. Simple bone cyst
H. Trauma
I. Rheumatoid arthritis
J. Osteoid osteoma
K. Ewing's tumour
L. Osteoporosis
M. Chondrosarcoma
N. Echondroma
O. Osteoclastoma
A

A. Osteochondroma

The commonest bony tumour. M:F = 3:1.
Usually around the knee, painless and slow growing. There may be a history of trauma such as a fracture.

234
Q
A 15 year old girl shows you a small lump on her upper arm on routine examination. She says the lump has been present for a couple of years and has slowly moved down, away from her shoulder.
A. Osteochondroma
B. Osteosarcoma
C. Fibrous dysplasia
D. Metastases
E. Osteitis
F. Osteoarthritis
G. Simple bone cyst
H. Trauma
I. Rheumatoid arthritis
J. Osteoid osteoma
K. Ewing's tumour
L. Osteoporosis
M. Chondrosarcoma
N. Echondroma
O. Osteoclastoma
A

G. Simple bone cyst

235
Q
A 50 year old lady presents with pain in her jaw. She suffers from Paget’s disease. A ‘sunburst’ appearance is seen on X-ray along with a lifted periosteum (Codman’s triangle).
A. Osteochondroma
B. Osteosarcoma
C. Fibrous dysplasia
D. Metastases
E. Osteitis
F. Osteoarthritis
G. Simple bone cyst
H. Trauma
I. Rheumatoid arthritis
J. Osteoid osteoma
K. Ewing's tumour
L. Osteoporosis
M. Chondrosarcoma
N. Echondroma
O. Osteoclastoma
A

B. Osteosarcoma

236
Q
An 8 year old boy is brought to his GP by his parents with pain in his hips and a fever. Blood results demonstrate a raised ESR and biopsy histology shows droplets of glycogen in the cytoplasm of small round cells in the pelvic bones.
A. Osteochondroma
B. Osteosarcoma
C. Fibrous dysplasia
D. Metastases
E. Osteitis
F. Osteoarthritis
G. Simple bone cyst
H. Trauma
I. Rheumatoid arthritis
J. Osteoid osteoma
K. Ewing's tumour
L. Osteoporosis
M. Chondrosarcoma
N. Echondroma
O. Osteoclastoma
A

K. Ewing’s tumour

237
Q
60 year old life long smoker presents to her GP with a 10 day history of worsening shortness of breath, cough and wheeze. Her cough produces a greenish sputum. On examination she is tachypnoeic and very cyanosed. Auscultation of her chest reveals a harsh polyphonic wheeze and signs of a right lower lobe pneumonia. This lady has developed type II respiratory failure secondary to what pulmonary disease?
A. Cystic fibrosis
B. Cryptogenic fibrosing alveolitis
C. Mycoplasma pneumonia
D. Acute asthma
E. Small cell carcinoma
F. Pulmonary oedema
G. Tuberculosis
H. Emphysema
I. Pulmonary embolism
J. Thymoma
K. Chronic bronchitis
L. Pneumococcal pneumonia
M. Squamous cell carcinoma
A

K. Chronic bronchitis

238
Q
70 year old man presents to his GP with a four day history of haemoptysis. He has noticed he has been loosing weight over the last 4 months and has felt tired and unwell. On examination he has bilateral ptosis and proximal weakness in the limbs which improves on repeated testing
A. Cystic fibrosis
B. Cryptogenic fibrosing alveolitis
C. Mycoplasma pneumonia
D. Acute asthma
E. Small cell carcinoma
F. Pulmonary oedema
G. Tuberculosis
H. Emphysema
I. Pulmonary embolism
J. Thymoma
K. Chronic bronchitis
L. Pneumococcal pneumonia
M. Squamous cell carcinoma
A

E. Small cell carcinoma

He has Eaton Lambert Syndrome which occurs in about 3% of people with small cell lung cancer.

239
Q
52 year old lady with ovarian cancer comes into A&amp;E with with a 10 hour history of haemoptysis associated with dysponea and pleuritic pain. Her chest X-ray is normal apart from a wedge shaped infarct peripherally on the right.
A. Cystic fibrosis
B. Cryptogenic fibrosing alveolitis
C. Mycoplasma pneumonia
D. Acute asthma
E. Small cell carcinoma
F. Pulmonary oedema
G. Tuberculosis
H. Emphysema
I. Pulmonary embolism
J. Thymoma
K. Chronic bronchitis
L. Pneumococcal pneumonia
M. Squamous cell carcinom
A

I. Pulmonary embolism

240
Q
45 year old fireman presents to chest clinic with a 1 year history of increasing shortness of breath. On examination you find that he is tachypnoeic and has gross clubbing of the fingers. Auscultation reveals bibasal fine end-inspiratory crackles
A. Cystic fibrosis
B. Cryptogenic fibrosing alveolitis
C. Mycoplasma pneumonia
D. Acute asthma
E. Small cell carcinoma
F. Pulmonary oedema
G. Tuberculosis
H. Emphysema
I. Pulmonary embolism
J. Thymoma
K. Chronic bronchitis
L. Pneumococcal pneumonia
M. Squamous cell carcinom
A

B. Cryptogenic fibrosing alveolitis

=idiopathic pulmonary fibrosis

241
Q
30 year old lady presents to her GP with a productive cough. The sputum is investigated and shows neutrophils and gram-positive cocci in pairs.
A. Cystic fibrosis
B. Cryptogenic fibrosing alveolitis
C. Mycoplasma pneumonia
D. Acute asthma
E. Small cell carcinoma
F. Pulmonary oedema
G. Tuberculosis
H. Emphysema
I. Pulmonary embolism
J. Thymoma
K. Chronic bronchitis
L. Pneumococcal pneumonia
M. Squamous cell carcinom
A

L. Pneumococcal pneumonia

242
Q
A 35 year old woman presents with weight loss and tiredness. Her GP examines her and finds that she also has a fine tremor and exopthalmos. Investigations: TSH <0.01, Free T4 36.0. There is high uptake on a technetium scan.
A. De Quervain's
B. Subacute thyroiditis
C. Thyroid storm
D. Hashimoto's thyroiditis
E. Postpartum thyroiditis
F. Grave's disease
G. Medullary carcinoma
H. Euthyroid state
I. Papillary carcinoma
A

F. Grave’s disease

243
Q
A 35 year old woman presents with weight loss and tiredness. Her GP examines her and finds that she also has a fine tremor and is sweaty. Investigations: TSH <0.01, Free T4 36.0. There is low uptake on a technetium scan. The aetiology could be viral or autoimmune.
A. De Quervain's
B. Subacute thyroiditis
C. Thyroid storm
D. Hashimoto's thyroiditis
E. Postpartum thyroiditis
F. Grave's disease
G. Medullary carcinoma
H. Euthyroid state
I. Papillary carcinoma
A

B. Subacute thyroiditis

The thyroid gland is damaged and initially releases thyroid hormones, giving free T4 and thyrotoxicosis symptoms for a month or so. Eventually the thyroid hormones run out and so levels fall leading to hypothyroidism. There is damage to the cells so they do not readily take up iodine.

If it was caused by viral (De Quervains) then there would be fever and painful thyroid and the hypothyroidism can last up to 2 months. If it were caused by autoimmune (Hashimotos) the hypothyroidism would be chronic.

244
Q
A 42 year old woman presents to her GP suffering from mild depression. Her thyroid function is investigated: TSH 6.5, Free T4 5.6
A. De Quervain's
B. Subacute thyroiditis
C. Thyroid storm
D. Hashimoto's thyroiditis
E. Postpartum thyroiditis
F. Grave's disease
G. Medullary carcinoma
H. Euthyroid state
I. Papillary carcinoma
A

D. Hashimoto’s thyroiditis

245
Q
An 18 year old man notices a lump on his neck and goes to his GP. As well as the lump, the GP discovers cervical lymphadenopathy. There is no family history of any endocrine disorder, nor is he suffering from any other illness. Thyroglobulin is 140.
A. De Quervain's
B. Subacute thyroiditis
C. Thyroid storm
D. Hashimoto's thyroiditis
E. Postpartum thyroiditis
F. Grave's disease
G. Medullary carcinoma
H. Euthyroid state
I. Papillary carcinoma
A

I. Papillary carcinoma

246
Q
A 27 year old man goes to his GP with a 3 day history of fever, tiredness and neck pain. His ESR is raised. TSH 0.12, Free T4 32. After 2 weeks his symptoms have resolved. His GP repeats his thyroid function tests: TSH 5.9, Free T4 8.4. After a further week his thyroid function has returned to completely normal levels.
A. De Quervain's
B. Subacute thyroiditis
C. Thyroid storm
D. Hashimoto's thyroiditis
E. Postpartum thyroiditis
F. Grave's disease
G. Medullary carcinoma
H. Euthyroid state
I. Papillary carcinoma
A

A. De Quervain’s

247
Q
Benign hemispherical neoplasms projecting from the endometrial mucosa into the uterine cavity. They can cause uterine bleeding and infertility and are commonest at the time of menopause
A. Teratoma (Mature)
B. Teratoma (Immature)
C. Endometrioid Tumour
D. Endometrial Adenocarcinoma
E. Cervical Squamous Cell Carcinoma
F. Vaginal Squamous Cell Carcinoma
G. Serous Tumour
H. Mucinous Tumour
I. Cervical Intraepithelial Neoplasia (CIN)
J. Endometrial Polyps
A

J. Endometrial Polyps

248
Q
Neoplasms found in young women as ovarian masses, usually unilateral and most often on the right. They are usually filled with sebaceous secretion and matted hair and may have a nodular projection from which teeth protrude.
A. Teratoma (Mature)
B. Teratoma (Immature)
C. Endometrioid Tumour
D. Endometrial Adenocarcinoma
E. Cervical Squamous Cell Carcinoma
F. Vaginal Squamous Cell Carcinoma
G. Serous Tumour
H. Mucinous Tumour
I. Cervical Intraepithelial Neoplasia (CIN)
J. Endometrial Polyps
A

A. Teratoma (Mature)

249
Q
The most common cancerous neoplasm of the female genital tract. It appears most frequently between the ages of 55 and 65, with obesity, diabetes, hypertension and infertility being the major risk factors.
A. Teratoma (Mature)
B. Teratoma (Immature)
C. Endometrioid Tumour
D. Endometrial Adenocarcinoma
E. Cervical Squamous Cell Carcinoma
F. Vaginal Squamous Cell Carcinoma
G. Serous Tumour
H. Mucinous Tumour
I. Cervical Intraepithelial Neoplasia (CIN)
J. Endometrial Polyps
A

D. Endometrial Adenocarcinoma

250
Q
Neoplasms found in women aged between 30 and 40 as ovarian masses, usually unilateral. They are usually benign (90%) and are often the largest ovarian neoplasm.
A. Teratoma (Mature)
B. Teratoma (Immature)
C. Endometrioid Tumour
D. Endometrial Adenocarcinoma
E. Cervical Squamous Cell Carcinoma
F. Vaginal Squamous Cell Carcinoma
G. Serous Tumour
H. Mucinous Tumour
I. Cervical Intraepithelial Neoplasia (CIN)
J. Endometrial Polyps
A

H. Mucinous Tumour

They account for 10-15% of all ovarian tumours and are rarely malignant. They can be very large and are usually unilateral

251
Q
A common neoplasm which can be detected by cytological examination long before any gross abnormality are seen. Major risk factors include early age at first intercourse, multiple sexual partners and persistent infection by human papillomavirus (HPV). Histologically, there is mild hyperchromasia and finely stippled chromatin pattern.
A. Teratoma (Mature)
B. Teratoma (Immature)
C. Endometrioid Tumour
D. Endometrial Adenocarcinoma
E. Cervical Squamous Cell Carcinoma
F. Vaginal Squamous Cell Carcinoma
G. Serous Tumour
H. Mucinous Tumour
I. Cervical Intraepithelial Neoplasia (CIN)
J. Endometrial Polyps
A

I. Cervical Intraepithelial Neoplasia (CIN)

252
Q
Squamous epithelium mixed with intestinal epithelium
A. Serous cystadenocarcinoma
B. Mature cystic teratoma
C. Thecoma
D. Dysgerminoma
E. Sertoli-Leydig tumour
F. Krukenberg tumour
G. Clear cell tumour
H. Granulosa cell tumours
I. Mucinous cystadenocarcinoma
J. Immature teratoma
K. Fibroma
A

B. Mature cystic teratoma

253
Q
Fibrous tissue containing spindle cells and lipid
A. Serous cystadenocarcinoma
B. Mature cystic teratoma
C. Thecoma
D. Dysgerminoma
E. Sertoli-Leydig tumour
F. Krukenberg tumour
G. Clear cell tumour
H. Granulosa cell tumours
I. Mucinous cystadenocarcinoma
J. Immature teratoma
K. Fibroma
A

C. Thecoma

Contain bundles of spindle cells and lipid

254
Q
Malignant cells surrounded by serous fluid and Psammoma bodies
A. Serous cystadenocarcinoma
B. Mature cystic teratoma
C. Thecoma
D. Dysgerminoma
E. Sertoli-Leydig tumour
F. Krukenberg tumour
G. Clear cell tumour
H. Granulosa cell tumours
I. Mucinous cystadenocarcinoma
J. Immature teratoma
K. Fibroma
A

A. Serous cystadenocarcinoma

255
Q
Malignant signet ring cells containing mucin
A. Serous cystadenocarcinoma
B. Mature cystic teratoma
C. Thecoma
D. Dysgerminoma
E. Sertoli-Leydig tumour
F. Krukenberg tumour
G. Clear cell tumour
H. Granulosa cell tumours
I. Mucinous cystadenocarcinoma
J. Immature teratoma
K. Fibroma
A

F. Krukenberg tumour

256
Q
Germ cells mixed with lymphocytes
A. Serous cystadenocarcinoma
B. Mature cystic teratoma
C. Thecoma
D. Dysgerminoma
E. Sertoli-Leydig tumour
F. Krukenberg tumour
G. Clear cell tumour
H. Granulosa cell tumours
I. Mucinous cystadenocarcinoma
J. Immature teratoma
K. Fibroma
A

D. Dysgerminoma

257
Q
Germ cell tumour originating in testis that is radiosensitive and classically presents in the 4th decade.
A. Leydig cell tumour
B. Teratoma
C. Acute myeloid leukaemia (AML)
D. Yolk sac (endodermal sinus) tumour
E. Embryonal carcinoma
F. Sertoli cell tumour
G. Diffuse large B cell lymphoma (DLBCL)
H. Acute lymphoblastic leukaemia/lymphoma (ALL)
I. Choriocarcinoma
J. Seminoma
A

J. Seminoma

258
Q
Very aggressive tumour producing HCG and AFP; neoplastic cells are anaplastic.
A. Leydig cell tumour
B. Teratoma
C. Acute myeloid leukaemia (AML)
D. Yolk sac (endodermal sinus) tumour
E. Embryonal carcinoma
F. Sertoli cell tumour
G. Diffuse large B cell lymphoma (DLBCL)
H. Acute lymphoblastic leukaemia/lymphoma (ALL)
I. Choriocarcinoma
J. Seminoma
A

E. Embryonal carcinoma

259
Q
Very aggressive HCG-producing tumour composed of cytotrophoblast and syncytiotrophoblast cells that metastasizes early.	
A. Leydig cell tumour
B. Teratoma
C. Acute myeloid leukaemia (AML)
D. Yolk sac (endodermal sinus) tumour
E. Embryonal carcinoma
F. Sertoli cell tumour
G. Diffuse large B cell lymphoma (DLBCL)
H. Acute lymphoblastic leukaemia/lymphoma (ALL)
I. Choriocarcinoma
J. Seminoma
A

I. Choriocarcinoma

260
Q
Nonseminomatous germ cell tumour that has elements derived from all 3 germ cell layers
A. Leydig cell tumour
B. Teratoma
C. Acute myeloid leukaemia (AML)
D. Yolk sac (endodermal sinus) tumour
E. Embryonal carcinoma
F. Sertoli cell tumour
G. Diffuse large B cell lymphoma (DLBCL)
H. Acute lymphoblastic leukaemia/lymphoma (ALL)
I. Choriocarcinoma
J. Seminoma
A

B. Teratoma

261
Q
Commonest malignant cause of testicular mass in those aged under 5.
A. Leydig cell tumour
B. Teratoma
C. Acute myeloid leukaemia (AML)
D. Yolk sac (endodermal sinus) tumour
E. Embryonal carcinoma
F. Sertoli cell tumour
G. Diffuse large B cell lymphoma (DLBCL)
H. Acute lymphoblastic leukaemia/lymphoma (ALL)
I. Choriocarcinoma
J. Seminoma
A

H. Acute lymphoblastic leukaemia/lymphoma (ALL)

262
Q
Commonest malignant cause of testicular mass in those aged 60
A. Leydig cell tumour
B. Teratoma
C. Acute myeloid leukaemia (AML)
D. Yolk sac (endodermal sinus) tumour
E. Embryonal carcinoma
F. Sertoli cell tumour
G. Diffuse large B cell lymphoma (DLBCL)
H. Acute lymphoblastic leukaemia/lymphoma (ALL)
I. Choriocarcinoma
J. Seminoma
A

G. Diffuse large B cell lymphoma (DLBCL)

263
Q
A 29-year-old lady presented with painful ulceration of the vulval skin. Histologically, there were intraepithelial blisters, intranuclear viral inclusions and eosinophilic cytoplasmic swelling.
A. Neisseria gonorrheae
B. Gardnerella vaginalis
C. Herpes simplex
D. Cervical Intraepithelial Neoplasia III
E. Chlamydia
F. Cervical Intraepithelial Neoplasia I
G. Candida albicans
H. Cervical polyps
I. Cervical Intraepithelial Neoplasia II
J. Cervical Microglandular Hyperplasia
K. Treponema pallidum (syphilis)
L. Trichomonas vaginalis
M. Lymphogranuloma venereum
N. Granuloma Inguinale
O. Herpes zoster
P. Group B Streptococcus
A

C. Herpes simplex

264
Q
A 42-year-old lady presented to a clinic for a smear test but explained that due to living abroad she had not had one for 8 years. The smear showed severe dyskaryosis and she was referred for colposcopic examination, which revealed dysplastic changes in the full thickness of the cervical epithelium.
A. Neisseria gonorrheae
B. Gardnerella vaginalis
C. Herpes simplex
D. Cervical Intraepithelial Neoplasia III
E. Chlamydia
F. Cervical Intraepithelial Neoplasia I
G. Candida albicans
H. Cervical polyps
I. Cervical Intraepithelial Neoplasia II
J. Cervical Microglandular Hyperplasia
K. Treponema pallidum (syphilis)
L. Trichomonas vaginalis
M. Lymphogranuloma venereum
N. Granuloma Inguinale
O. Herpes zoster
P. Group B Streptococcus
A

D. Cervical Intraepithelial Neoplasia III

265
Q
A young lady is found to have chronic irritation and inflammation of the vulva. A pap smear and use of a silver stain reveals fungi within the keratin layer and superficial epithelium.
A. Neisseria gonorrheae
B. Gardnerella vaginalis
C. Herpes simplex
D. Cervical Intraepithelial Neoplasia III
E. Chlamydia
F. Cervical Intraepithelial Neoplasia I
G. Candida albicans
H. Cervical polyps
I. Cervical Intraepithelial Neoplasia II
J. Cervical Microglandular Hyperplasia
K. Treponema pallidum (syphilis)
L. Trichomonas vaginalis
M. Lymphogranuloma venereum
N. Granuloma Inguinale
O. Herpes zoster
P. Group B Streptococcus
A

G. Candida albicans

266
Q
A 22-year-old lady presented with a vaginal discharge. Gram staining revealed “Clue cells” surrounded by rods, that were “Gram variable”.
A. Neisseria gonorrheae
B. Gardnerella vaginalis
C. Herpes simplex
D. Cervical Intraepithelial Neoplasia III
E. Chlamydia
F. Cervical Intraepithelial Neoplasia I
G. Candida albicans
H. Cervical polyps
I. Cervical Intraepithelial Neoplasia II
J. Cervical Microglandular Hyperplasia
K. Treponema pallidum (syphilis)
L. Trichomonas vaginalis
M. Lymphogranuloma venereum
N. Granuloma Inguinale
O. Herpes zoster
P. Group B Streptococcus
A

B. Gardnerella vaginalis

267
Q
A pap smear taken from a chronic granulomatous ulcer shows a necrotic centre, periarteritis and endarteritis obliterans and an intense peripheral cellular infiltrate consisting mainly of mononuclear cells and giant cells.
A. Neisseria gonorrheae
B. Gardnerella vaginalis
C. Herpes simplex
D. Cervical Intraepithelial Neoplasia III
E. Chlamydia
F. Cervical Intraepithelial Neoplasia I
G. Candida albicans
H. Cervical polyps
I. Cervical Intraepithelial Neoplasia II
J. Cervical Microglandular Hyperplasia
K. Treponema pallidum (syphilis)
L. Trichomonas vaginalis
M. Lymphogranuloma venereum
N. Granuloma Inguinale
O. Herpes zoster
P. Group B Streptococcus
A

K. Treponema pallidum (syphilis)

268
Q
The peak incidence of cervical cancer occurs at what age?
A. Invasive cervical cancer
B. CIN III
C. Ovarian cancer
D. CIN II
E. 65 years
F. Squamous to glandular epitheliem
G. 30 - 40 years
H. Glandular to squamous epithelium
I. 50-60 years
J. CIN I
K. 45 years
A

G. 30 - 40 years

269
Q
The metaplasia that occurs in the transformation zone involves which cell-types?
A. Invasive cervical cancer
B. CIN III
C. Ovarian cancer
D. CIN II
E. 65 years
F. Squamous to glandular epitheliem
G. 30 - 40 years
H. Glandular to squamous epithelium
I. 50-60 years
J. CIN I
K. 45 years
A

H. Glandular to squamous epithelium

270
Q
A 27 year old woman undergoes a routine Pap test. Histology shows increased squamous epithelium with atypical cells showing koilocytosis (nuclear enlargement with perinuclear halo = clear area around nucleus). Follow-up colposcopy shows hyperchromatic nuclei present in the lower 1/3 of the epithelial layer from the basement membrane.
A. Invasive cervical cancer
B. CIN III
C. Ovarian cancer
D. CIN II
E. 65 years
F. Squamous to glandular epitheliem
G. 30 - 40 years
H. Glandular to squamous epithelium
I. 50-60 years
J. CIN I
K. 45 years
A

J. CIN I

271
Q
Histology from the transformation zone in a 40 year old woman demonstrates increased squamous cells with abnormal features through the full thickness of the epithelium. The basement membrane has been breached.
A. Invasive cervical cancer
B. CIN III
C. Ovarian cancer
D. CIN II
E. 65 years
F. Squamous to glandular epitheliem
G. 30 - 40 years
H. Glandular to squamous epithelium
I. 50-60 years
J. CIN I
K. 45 years
A

A. Invasive cervical cancer

272
Q
NHS Cervial Screening Programme currently screens women with normal Pap tests up to what age?
A. Invasive cervical cancer
B. CIN III
C. Ovarian cancer
D. CIN II
E. 65 years
F. Squamous to glandular epitheliem
G. 30 - 40 years
H. Glandular to squamous epithelium
I. 50-60 years
J. CIN I
K. 45 years
A

E. 65 years

273
Q
A 29 year old lady presented with painful ulceration of the vulval skin. Histologically, there were intraepithelial blisters, intranuclear viral inclusions and eosinophilic cytoplasmic swelling
A. Candida albicans
B. Mild dyskaryosis
C. Granuloma Inguinale
D. Trichomonas vaginalis
E. Cervical Intraepithelial Neoplasia III
F. Gardnerella vaginalis
G. Lymphogranuloma venereum
H. Cervical Microglandular Hyperplasia
I. Treponema pallidum (syphilis)
J. Neisseria gonorrheae
K. Cervical Intraepithelial Neoplasia I
L. Group B Streptococcus
M. Chlamydia
N. Severe dyskaryosis
O. Herpes simplex
A

O. Herpes simplex

274
Q
A 42 year old lady presented to a clinic for a smear test but explained that due to living abroad she had not had one for 8 years. The smear showed severe dyskaryosis and she was referred for colposcopic examination, which revealed dysplastic changes in the full thickness of the cervical epithelium.
A. Candida albicans
B. Mild dyskaryosis
C. Granuloma Inguinale
D. Trichomonas vaginalis
E. Cervical Intraepithelial Neoplasia III
F. Gardnerella vaginalis
G. Lymphogranuloma venereum
H. Cervical Microglandular Hyperplasia
I. Treponema pallidum (syphilis)
J. Neisseria gonorrheae
K. Cervical Intraepithelial Neoplasia I
L. Group B Streptococcus
M. Chlamydia
N. Severe dyskaryosis
O. Herpes simplex
A

E. Cervical Intraepithelial Neoplasia III

275
Q
A young lady is found to have chronic irritation and inflammation of the vulva. A pap smear and use of a silver stain reveals fungi within the keratin layer and superficial epithelium.
A. Candida albicans
B. Mild dyskaryosis
C. Granuloma Inguinale
D. Trichomonas vaginalis
E. Cervical Intraepithelial Neoplasia III
F. Gardnerella vaginalis
G. Lymphogranuloma venereum
H. Cervical Microglandular Hyperplasia
I. Treponema pallidum (syphilis)
J. Neisseria gonorrheae
K. Cervical Intraepithelial Neoplasia I
L. Group B Streptococcus
M. Chlamydia
N. Severe dyskaryosis
O. Herpes simplex
A

A. Candida albicans

276
Q
A 22 year old lady presented for her routine smear test. The test showed a slight increase in DNA staining, and also a slightly larger variation in size of nuclei. Further questioning revealed she was a single mother living with her boyfriend, and had had multiple sexual partners since her first sexual contact at age 16.
A. Candida albicans
B. Mild dyskaryosis
C. Granuloma Inguinale
D. Trichomonas vaginalis
E. Cervical Intraepithelial Neoplasia III
F. Gardnerella vaginalis
G. Lymphogranuloma venereum
H. Cervical Microglandular Hyperplasia
I. Treponema pallidum (syphilis)
J. Neisseria gonorrheae
K. Cervical Intraepithelial Neoplasia I
L. Group B Streptococcus
M. Chlamydia
N. Severe dyskaryosis
O. Herpes simplex
A

B. Mild dyskaryosis

277
Q
A pap smear taken from a chronic granulomatous ulcer shows a necrotic centre, periarteritis and endarteritis obliterans and an intense peripheral cellular infiltrate consisting mainly of mononuclear cells and giant cells.
A. Candida albicans
B. Mild dyskaryosis
C. Granuloma Inguinale
D. Trichomonas vaginalis
E. Cervical Intraepithelial Neoplasia III
F. Gardnerella vaginalis
G. Lymphogranuloma venereum
H. Cervical Microglandular Hyperplasia
I. Treponema pallidum (syphilis)
J. Neisseria gonorrheae
K. Cervical Intraepithelial Neoplasia I
L. Group B Streptococcus
M. Chlamydia
N. Severe dyskaryosis
O. Herpes simplex
A

I. Treponema pallidum (syphilis)

278
Q
Single most useful first-line cytological investigation of an impalpable breast lump seen on a mammogram.
A. Fluid cytology of alveolar washings
B. Percutaneous FNA biopsy
C. Core biopsy
D. Cone biopsy
E. Exfoliative (brush) cytology
F. Stereotactic radiographic cytological sampling method
G. Endometrial tissue sampling
H. Cystoscopy
I. Fine needle aspirate
A

F. Stereotactic radiographic cytological sampling method

279
Q
Single most useful first-line cytological investigation for the confirmation of the benign status of an ovarian cyst
A. Fluid cytology of alveolar washings
B. Percutaneous FNA biopsy
C. Core biopsy
D. Cone biopsy
E. Exfoliative (brush) cytology
F. Stereotactic radiographic cytological sampling method
G. Endometrial tissue sampling
H. Cystoscopy
I. Fine needle aspirate.
A

I. Fine needle aspirate.

280
Q
Single most useful first-line cytological investigation for the detection of pneumocystis carinii in an immunosuppressed patient.
A. Fluid cytology of alveolar washings
B. Percutaneous FNA biopsy
C. Core biopsy
D. Cone biopsy
E. Exfoliative (brush) cytology
F. Stereotactic radiographic cytological sampling method
G. Endometrial tissue sampling
H. Cystoscopy
I. Fine needle aspirate.
A

A. Fluid cytology of alveolar washings

281
Q
Single most useful first-line cytological investigation of a palpable breast lump in a clinic setting.
A. Fluid cytology of alveolar washings
B. Percutaneous FNA biopsy
C. Core biopsy
D. Cone biopsy
E. Exfoliative (brush) cytology
F. Stereotactic radiographic cytological sampling method
G. Endometrial tissue sampling
H. Cystoscopy
I. Fine needle aspirate.
A

I. Fine needle aspirate.

282
Q
Single most useful first-line cytological investigation for screening for cervical dysplasia
A. Fluid cytology of alveolar washings
B. Percutaneous FNA biopsy
C. Core biopsy
D. Cone biopsy
E. Exfoliative (brush) cytology
F. Stereotactic radiographic cytological sampling method
G. Endometrial tissue sampling
H. Cystoscopy
I. Fine needle aspirate.
A

E. Exfoliative (brush) cytology

283
Q

FNA
A. Leads to pain and trauma to the patient.
B. Requires a hospital stay, possible general anaesthetic and leads to pain and trauma to the patient.
C. Has an 85% positive predictive value for malignant cytopathological diagnosis.
D. Has a 100% positive predictive value for malignant cytopathological diagnosis.
E. Has a 10% false positive rate
F. Does not allow patient counselling prior to definitive treatment.

A

D. Has a 100% positive predictive value for malignant cytopathological diagnosis.

284
Q

Needle core biopsy
A. Leads to pain and trauma to the patient.
B. Requires a hospital stay, possible general anaesthetic and leads to pain and trauma to the patient.
C. Has an 85% positive predictive value for malignant cytopathological diagnosis.
D. Has a 100% positive predictive value for malignant cytopathological diagnosis.
E. Has a 10% false positive rate
F. Does not allow patient counselling prior to definitive treatment.

A

A. Leads to pain and trauma to the patient.

285
Q

Surgical biopsy of lesions
A. Leads to pain and trauma to the patient.
B. Requires a hospital stay, possible general anaesthetic and leads to pain and trauma to the patient.
C. Has an 85% positive predictive value for malignant cytopathological diagnosis.
D. Has a 100% positive predictive value for malignant cytopathological diagnosis.
E. Has a 10% false positive rate
F. Does not allow patient counselling prior to definitive treatment.

A

B. Requires a hospital stay, possible general anaesthetic and leads to pain and trauma to the patient.

286
Q

Intra-operative frozen section diagnosis
A. Leads to pain and trauma to the patient.
B. Requires a hospital stay, possible general anaesthetic and leads to pain and trauma to the patient.
C. Has an 85% positive predictive value for malignant cytopathological diagnosis.
D. Has a 100% positive predictive value for malignant cytopathological diagnosis.
E. Has a 10% false positive rate
F. Does not allow patient counselling prior to definitive treatment.

A

F. Does not allow patient counselling prior to definitive treatment.

287
Q

What is the most common type of thyroid cancer?

A

Papillary carcinoma

-Optically clear nuclei, intranuclear inclusions, psammoma bodies

288
Q
Which type of thyroid cancer derives from parafollicular C cells?
A. Medullary 
B. Papillary 
C. Anaplastic 
D. Follicular
A

A. Medullary

289
Q
A 35 year old woman was brought to A&amp;E with a severe headache. She was lucid and mentioned that she suffered from frequent headaches but that this one felt like a hammer-blow. She was photophobic and nauseous with slight neck stiffness. Her pulse rate was 75bpm, and BP was 130/80.
A. Subarachnoid haemorrhage
B. Meningitis
C. Transient hypotension
D. Brain stem infarction
E. Pontine haemorrhage
F. Panic attack
G. Subdural haemorrhage
H. Cerebral embolus
I. Alzheimer’s disease
J. Cerebellar stroke
K. Transient Ischaemic attack
L. Opiate overdose
M. Extradural haemorrhage
N. Temporal arteritis
O. Raised intracranial pressure
P. Vascular dementia
A

A. Subarachnoid haemorrhage

290
Q
A 70 year old woman goes to her GP accompanied by her daughter who looks after her. She has started undressing and wandering the streets in the night and she is aggressive and argumentative. She had a stroke 3 years ago and recovered well with only some transient ischaemic attacks recorded since. A CT scan has since shown areas of scattering of low density in the white matter.
A. Subarachnoid haemorrhage
B. Meningitis
C. Transient hypotension
D. Brain stem infarction
E. Pontine haemorrhage
F. Panic attack
G. Subdural haemorrhage
H. Cerebral embolus
I. Alzheimer’s disease
J. Cerebellar stroke
K. Transient Ischaemic attack
L. Opiate overdose
M. Extradural haemorrhage
N. Temporal arteritis
O. Raised intracranial pressure
P. Vascular dementia
A

P. Vascular dementia

291
Q
An 86 year old man is admitted to A&amp;E having collapsed at his home. He is unconscious and a couple of days later he is still deeply unconscious. His pupils are pin-point and their reaction to light is difficult to see clearly.
A. Subarachnoid haemorrhage
B. Meningitis
C. Transient hypotension
D. Brain stem infarction
E. Pontine haemorrhage
F. Panic attack
G. Subdural haemorrhage
H. Cerebral embolus
I. Alzheimer’s disease
J. Cerebellar stroke
K. Transient Ischaemic attack
L. Opiate overdose
M. Extradural haemorrhage
N. Temporal arteritis
O. Raised intracranial pressure
P. Vascular dementia
A

D. Brain stem infarction

292
Q
A 20 year old man was taken to A&amp;E after a pub brawl where he was stamped on whilst trying to break up a fight. He remained lucid for approximately 3 hours after the incident and then slipped in and out of consciousness with hemiparesis afterwards. He had a deep laceration on the side of his head. A CT scan showed high attenuation adjacent to the skull and a midline shift.
A. Subarachnoid haemorrhage
B. Meningitis
C. Transient hypotension
D. Brain stem infarction
E. Pontine haemorrhage
F. Panic attack
G. Subdural haemorrhage
H. Cerebral embolus
I. Alzheimer’s disease
J. Cerebellar stroke
K. Transient Ischaemic attack
L. Opiate overdose
M. Extradural haemorrhage
N. Temporal arteritis
O. Raised intracranial pressure
P. Vascular dementia
A

M. Extradural haemorrhage

293
Q
A 42 year old lady went to her GP in a frightened, agitated and still dazed state as the previous day she had what she described as a ‘strange migraine’ but without the headache when, she said, she suddenly could not see the TV. She said she thought it affected just the one eye and that she felt disorientated and unable to speak what she wanted to say. This lasted approximately 2-3 hours and then slowly started to clear. She was tachycardic with hot, sweaty palms.
A. Subarachnoid haemorrhage
B. Meningitis
C. Transient hypotension
D. Brain stem infarction
E. Pontine haemorrhage
F. Panic attack
G. Subdural haemorrhage
H. Cerebral embolus
I. Alzheimer’s disease
J. Cerebellar stroke
K. Transient Ischaemic attack
L. Opiate overdose
M. Extradural haemorrhage
N. Temporal arteritis
O. Raised intracranial pressure
P. Vascular dementia
A

K. Transient Ischaemic attack

294
Q
Occlusion of this cerebral vessel can cause weakness and numbness in the contralateral lower limb and similar but milder symptoms in the contralateral upper limb.
A. Subarachnoid haemorrhage
B. Posterior cerebral artery
C. Stroke
D. Amyloidosis
E. Extradural haemorrhage
F. Viral meningitis
G. Berry's aneurysm
H. Vertebrobasilar circulation
I. Hydrocephalus
J. Transient ischaemic attack (TIA)
K. Anterior cerebral artery
L. Bacterial meningitis
M. Middle cerebral artery
N. Lacunar infarcts
A

K. Anterior cerebral artery

295
Q
A 35-year old patient presented to A&amp;E complaining of severe headache and vomiting. On examination, there is neck stiffness; fundoscopy reveals preretinal (subhyaloid) haemorrhages; CSF is spun down and shows xanthochromia. You notice the patient has palpable kidneys bilaterally.
A. Subarachnoid haemorrhage
B. Posterior cerebral artery
C. Stroke
D. Amyloidosis
E. Extradural haemorrhage
F. Viral meningitis
G. Berry's aneurysm
H. Vertebrobasilar circulation
I. Hydrocephalus
J. Transient ischaemic attack (TIA)
K. Anterior cerebral artery
L. Bacterial meningitis
M. Middle cerebral artery
N. Lacunar infarcts
A

A. Subarachnoid haemorrhage

296
Q
This group of ischaemic brain lesions typically affect the basal ganglia, internal capsule, thalamus and pons and are smaller than 1cm.
A. Subarachnoid haemorrhage
B. Posterior cerebral artery
C. Stroke
D. Amyloidosis
E. Extradural haemorrhage
F. Viral meningitis
G. Berry's aneurysm
H. Vertebrobasilar circulation
I. Hydrocephalus
J. Transient ischaemic attack (TIA)
K. Anterior cerebral artery
L. Bacterial meningitis
M. Middle cerebral artery
N. Lacunar infarcts
A

N. Lacunar infarcts

297
Q
This structural abnormality of cerebral vessels usually affects the branching points of the circle or Willis and is a risk of regional infarcts in young patients.
A. Subarachnoid haemorrhage
B. Posterior cerebral artery
C. Stroke
D. Amyloidosis
E. Extradural haemorrhage
F. Viral meningitis
G. Berry's aneurysm
H. Vertebrobasilar circulation
I. Hydrocephalus
J. Transient ischaemic attack (TIA)
K. Anterior cerebral artery
L. Bacterial meningitis
M. Middle cerebral artery
N. Lacunar infarcts
A

G. Berry’s aneurysm

298
Q
Transtentorial herniations can potentially compromise the sufficiency this particular part of the cerebral circulation and cause occipital lobe infarction.
A. Subarachnoid haemorrhage
B. Posterior cerebral artery
C. Stroke
D. Amyloidosis
E. Extradural haemorrhage
F. Viral meningitis
G. Berry's aneurysm
H. Vertebrobasilar circulation
I. Hydrocephalus
J. Transient ischaemic attack (TIA)
K. Anterior cerebral artery
L. Bacterial meningitis
M. Middle cerebral artery
N. Lacunar infarcts
A

B. Posterior cerebral artery

299
Q
A back-office secretary in investment banking complains to her GP of intermittent attacks of tightness and pressure at the front of her head which sometimes radiates to the back and her neck. There is no associated vomiting or visual disturbance. She admits to feeling over-worked even though she got 6 months paid maternity leave.
A. Berry aneurysm
B. Sinusitis
C. Trigeminal neuralgia
D. Benign intracranial hypertension
E. Subarachnoid haemorrhage
F. Glaucoma
G. Migraine
H. Subdural haemorrhage
I. Tension headache
A

I. Tension headache

300
Q
A public school teacher is sitting at work eating a plougman’s when she starts to feel unwell and is unable to see properly, which lasts for 10 mins. Then one side of her head starts to throb and she begins to feel nauseous. Both bright lights and loud noises make her feel worse. 4 hours later it stops and she goes to sleep.
A. Berry aneurysm
B. Sinusitis
C. Trigeminal neuralgia
D. Benign intracranial hypertension
E. Subarachnoid haemorrhage
F. Glaucoma
G. Migraine
H. Subdural haemorrhage
I. Tension headache
A

G. Migraine

301
Q
25 year old obese woman goes to GP about worsening headaches. They are especially bad in the mornings and she also feels nauseous with some visual disturbances.
A. Berry aneurysm
B. Sinusitis
C. Trigeminal neuralgia
D. Benign intracranial hypertension
E. Subarachnoid haemorrhage
F. Glaucoma
G. Migraine
H. Subdural haemorrhage
I. Tension headache
A

D. Benign intracranial

302
Q
This condition is considered a medical emergency, but can lie undetected for years. Treatment is by micro-vascular clipping or occlusion.
A. Berry aneurysm
B. Sinusitis
C. Trigeminal neuralgia
D. Benign intracranial hypertension
E. Subarachnoid haemorrhage
F. Glaucoma
G. Migraine
H. Subdural haemorrhage
I. Tension headache
A

A. Berry aneurysm

303
Q
An elderly diabetic lady with an acute headache and associated blurred vision and vomiting. Pain is localised to the upper anterior region of her head.
A. Berry aneurysm
B. Sinusitis
C. Trigeminal neuralgia
D. Benign intracranial hypertension
E. Subarachnoid haemorrhage
F. Glaucoma
G. Migraine
H. Subdural haemorrhage
I. Tension headache
A

F. Glaucoma

304
Q
A 55 year old lady presents with a palpable lump in the left breast that is central with nipple discharge which is sometimes blood stained. FNA result shows that the lump has branching fibrovascular cores covered by cytologically bland epithelium
A. Lobular Carcinoma in situ
B. Fibroadenoma
C. Lymphoma
D. Invasive Carcinoma
E. Nipple adenoma
F. Gyaecomastia
G. Ductal Carcinoma in situ
H. Pagets Disease of the nipple
I. Duct Papilloma
A

I. Duct Papilloma

305
Q
A 60 year old lady presents with a hard palpable mass in the right breast. The mammography shows that the lump is calcified. FNA results show that the cells in the lump have pleomorphic nuclei and that the lump has a necrotic centre
A. Lobular Carcinoma in situ
B. Fibroadenoma
C. Lymphoma
D. Invasive Carcinoma
E. Nipple adenoma
F. Gyaecomastia
G. Ductal Carcinoma in situ
H. Pagets Disease of the nipple
I. Duct Papilloma
A

G. Ductal Carcinoma in situ

306
Q
A 23 year old lady comes to the breast clinic because she is worried of having breast cancer as her mother and grandmother both have had the disease. The mammogram finds no abnormality but you think you feel a number of multifocal lumps
A. Lobular Carcinoma in situ
B. Fibroadenoma
C. Lymphoma
D. Invasive Carcinoma
E. Nipple adenoma
F. Gyaecomastia
G. Ductal Carcinoma in situ
H. Pagets Disease of the nipple
I. Duct Papilloma
A

A. Lobular Carcinoma in situ

307
Q
A 45 year old lady is referred to the breast clinic with nipple retraction
A. Lobular Carcinoma in situ
B. Fibroadenoma
C. Lymphoma
D. Invasive Carcinoma
E. Nipple adenoma
F. Gyaecomastia
G. Ductal Carcinoma in situ
H. Pagets Disease of the nipple
A

D. Invasive Carcinoma

308
Q
A 30 year old woman is referred to the breast clinic with a red, roughened and ulcerated nipple.
A. Lobular Carcinoma in situ
B. Fibroadenoma
C. Lymphoma
D. Invasive Carcinoma
E. Nipple adenoma
F. Gyaecomastia
G. Ductal Carcinoma in situ
H. Pagets Disease of the nipple
A

H. Pagets Disease of the nipple

309
Q
55 years old grand-mother presents with a discrete unilateral breast lump. Biopsy reveals yellow, haemorrhagic tissue with flecks of calcification. She recalls being hit by a football across the chest whilst playing with her grandson
A. Acute mastitis
B. Lobular carcinoma in situ
C. Cyclic nodularity
D. Breast cyst
E. Paget’s disease
F. Invasive Carcinoma
G. Fibroadenoma
H. Traumatic fat necrosis
I. Mammary duct ectasis
A

H. Traumatic fat necrosis

310
Q
A 45 years old woman presents with multiple bilateral masses, each less than 10mm. FNA revealed serous, turbid fluid.
A. Acute mastitis
B. Lobular carcinoma in situ
C. Cyclic nodularity
D. Breast cyst
E. Paget’s disease
F. Invasive Carcinoma
G. Fibroadenoma
H. Traumatic fat necrosis
I. Mammary duct ectasis
A

D. Breast cyst

311
Q
A 30 years old woman presents with a unilateral mass. On examination it is firm, discrete, highly mobile and is less than 5cm in diameter.
A. Acute mastitis
B. Lobular carcinoma in situ
C. Cyclic nodularity
D. Breast cyst
E. Paget’s disease
F. Invasive Carcinoma
G. Fibroadenoma
H. Traumatic fat necrosis
I. Mammary duct ectasis
A

G. Fibroadenoma

312
Q
A 52 years old woman with a BMI of 30 presents with a 6 months history of bloody nipple discharge, and a firm palpable mass with signs of tethering on examination. She also complains of a 4 months history of back pain. Menarche started at the age of 9, and menopause started at 50 years old. Mammography revealed extensive calcification.
A. Acute mastitis
B. Lobular carcinoma in situ
C. Cyclic nodularity
D. Breast cyst
E. Paget’s disease
F. Invasive Carcinoma
G. Fibroadenoma
H. Traumatic fat necrosis
I. Mammary duct ectasis
A

F. Invasive Carcinoma

313
Q
A breastfeeding mother presents with a painfully enlarged breast. Examination reveals localised tenderness, erythema, skin ridging and pronounced cutaneous oedema, around cracked nipples
A. Acute mastitis
B. Lobular carcinoma in situ
C. Cyclic nodularity
D. Breast cyst
E. Paget’s disease
F. Invasive Carcinoma
G. Fibroadenoma
H. Traumatic fat necrosis
I. Mammary duct ectasis Mammography revealed extensive calcification.
A

A. Acute mastitis

314
Q
Cystic dilatation of duct during lactation caused by obstruction
A. gynaecomastia
B. fibrocystic change
C. fibroadenoma
D. lump
E. incidental finding on mammography
F. intraductal papilloma
G. mastitis
H. galactorrhoea
I. breathlessness
J. galactocoele
K. mammary duct ectasia
L. bloody discharge
A

J. galactocoele

315
Q
Infection of the breast that occurs during lactation
A. gynaecomastia
B. fibrocystic change
C. fibroadenoma
D. lump
E. incidental finding on mammography
F. intraductal papilloma
G. mastitis
H. galactorrhoea
I. breathlessness
J. galactocoele
K. mammary duct ectasia
L. bloody discharge
A

G. mastitis

316
Q
Occurs in males with liver disease
A. gynaecomastia
B. fibrocystic change
C. fibroadenoma
D. lump
E. incidental finding on mammography
F. intraductal papilloma
G. mastitis
H. galactorrhoea
I. breathlessness
J. galactocoele
K. mammary duct ectasia
L. bloody discharge
A

A. gynaecomastia

317
Q
Can occur in hypothyroidism due to raised TRH
A. gynaecomastia
B. fibrocystic change
C. fibroadenoma
D. lump
E. incidental finding on mammography
F. intraductal papilloma
G. mastitis
H. galactorrhoea
I. breathlessness
J. galactocoele
K. mammary duct ectasia
L. bloody discharge
A

H. galactorrhoea

318
Q
The commonest benign tumour of the female breast
A. gynaecomastia
B. fibrocystic change
C. fibroadenoma
D. lump
E. incidental finding on mammography
F. intraductal papilloma
G. mastitis
H. galactorrhoea
I. breathlessness
J. galactocoele
K. mammary duct ectasia
L. bloody discharge
A

C. fibroadenoma

319
Q
Included in the differential of bloody nipple discharge
A. gynaecomastia
B. fibrocystic change
C. fibroadenoma
D. lump
E. incidental finding on mammography
F. intraductal papilloma
G. mastitis
H. galactorrhoea
I. breathlessness
J. galactocoele
K. mammary duct ectasia
L. bloody discharge
A

F. intraductal papilloma

320
Q
Commonest presentation of invasive breast cancer
A. gynaecomastia
B. fibrocystic change
C. fibroadenoma
D. lump
E. incidental finding on mammography
F. intraductal papilloma
G. mastitis
H. galactorrhoea
I. breathlessness
J. galactocoele
K. mammary duct ectasia
L. bloody discharge
A

D. lump

321
Q
A 47 year-old woman visits her GP after having noticed a lump in the upper outer quadrant of her right breast. The lump is fixed and is not painful on palpation. She is also complaining of back pain.
A. Fibroadenoma
B. BRCA1 gene
C. Carcinoma-in-situ (ductal)
D. Fine needle aspirate
E. Nulliparity
F. Hormone replacement therapy
G. Carcinoma-in-situ (lobular)
H. Oestrogen-receptor positivity
I. Invasive breast carcinoma
J. Paget's disease of the nipple
K. Axillary lymph node metastases
A

I. Invasive breast carcinoma

322
Q
A 32 year-old woman presents to her GP with a lump on her left breast which she noticed 8 months ago. The lump is mobile and she noticed it grew more rapidly while she was pregnant with her fourth child.
A. Fibroadenoma
B. BRCA1 gene
C. Carcinoma-in-situ (ductal)
D. Fine needle aspirate
E. Nulliparity
F. Hormone replacement therapy
G. Carcinoma-in-situ (lobular)
H. Oestrogen-receptor positivity
I. Invasive breast carcinoma
J. Paget's disease of the nipple
K. Axillary lymph node metastases
A

A. Fibroadenoma

323
Q
A 56-year old female presents with a palpable mass in her left breast. Radiology shows 
A. Fibroadenoma
B. BRCA1 gene
C. Carcinoma-in-situ (ductal)
D. Fine needle aspirate
E. Nulliparity
F. Hormone replacement therapy
G. Carcinoma-in-situ (lobular)
H. Oestrogen-receptor positivity
I. Invasive breast carcinoma
J. Paget's disease of the nipple
K. Axillary lymph node
A

C. Carcinoma-in-situ (ductal)

324
Q
A 63 year-old female has noticed reddening around her nipple and describes the skin as scaly.
A. Fibroadenoma
B. BRCA1 gene
C. Carcinoma-in-situ (ductal)
D. Fine needle aspirate
E. Nulliparity
F. Hormone replacement therapy
G. Carcinoma-in-situ (lobular)
H. Oestrogen-receptor positivity
I. Invasive breast carcinoma
J. Paget's disease of the nipple
K. Axillary lymph node
A

J. Paget’s disease of the nipple

325
Q
Which feature, after histological confirmation of a breast carcinoma, would suggest a worse prognosis?
A. Fibroadenoma
B. BRCA1 gene
C. Carcinoma-in-situ (ductal)
D. Fine needle aspirate
E. Nulliparity
F. Hormone replacement therapy
G. Carcinoma-in-situ (lobular)
H. Oestrogen-receptor positivity
I. Invasive breast carcinoma
J. Paget's disease of the nipple
K. Axillary lymph node
A

K. Axillary lymph node

326
Q

What is a mallory denk body?

A

Inclusion found in the cytoplasm of liver cells. It is a sign of damaged intermediate filaments in hepatocytes

327
Q

Autoimmune hepatitis is more common in males or females?

A

More common in females

328
Q

Give three complications of liver cirrhosis

A
  1. portal HTN (shunting)
  2. Liver cell cancer
  3. Hepatic encephalopathy
329
Q
Spotty necrosis would be a histological feature of:
A. Acute hepatitis
B. Chronic hepatitis
C. Autoimmune hepatitis
D. Cirrhosis
A

A. Acute hepatitis

330
Q
In chronic hepatitis, the grade corresponds to:
A. Severity of inflammation 
B. Severity of fibrosis 
C. Severity of symptoms
D. Viral load
A

A. Severity of inflammation

331
Q
In chronic hepatitis, the stage corresponds to:
A. Severity of inflammation 
B. Severity of fibrosis 
C. Severity of symptoms
D. Viral load
A

B. Severity of fibrosis

332
Q
Bile duct loss associated with chronic inflammation:
A. Autoimmune hepatitis 
B. Viral hepatitis
C. Primary biliary cholangitis
D. Primary sclerosis cholangitis
A

C. Primary biliary cholangitis

333
Q
Periductal bile duct fibrosis leading to loss:
A. Autoimmune hepatitis 
B. Viral hepatitis
C. Primary biliary cholangitis
D. Primary sclerosis cholangitis
A

D. Primary sclerosis cholangitis

334
Q

What is haemochromatosis?

A

Genetically determined increase in gut absorption of iron leading to parenchymal damage to organs secondary to iron deposition (e.g. in hepatocytes)
Also known as bronzed diabetes

Gene on chromosome 6

335
Q

What is haemosiderosis?

A

Accumulation of iron in macrophages

336
Q
A positive rhodanine stain suggests: 
A. Haemochromatosis 
B. Amyloidosis 
C. Wilsons disease
D. Alpha 1 anti trypsin deficiency
A

C. Wilsons disease

Accumulation of copper due to failure of excretion by hepatocytes into the bile
Copper accumulates in the liver and CNS
A gene on chromosome 13

337
Q

Which antibodies are seen in autoimmune hepatitis?

A

Anti smooth muscle actin antibodies

338
Q
Alpha 1 anti trypsin deficiency can lead to which of the following?
A. Cirrhosis 
B. Hepatitis
C. Pneumonia
D. Haemosiderosis 
E. Primary biliary cholangitis
A

A and B

It is a failure to secrete alpha 1 antitrypsin but there is an excess in the hepatocytes = intra cytoplasmic inclusions due to the misfolded proteins

339
Q

Hepatic granulomas are seen in:
A. Primary biliary cholangitis
B. Primary sclerosing cholangitis

A

A. PBC

340
Q

List some causes of hepatic granulomas?

A

PBC
Drugs
TB
Sarcoid

341
Q
Which of the following is not a malignant liver tumour?
A. Haemangiosarcoma
B. Hepatoblastoma
C. Haemangioma 
D. Hepatocellular carcinoma
A

C. Haemangioma (most common benign tumour of the liver)

Other benign tumours of the liver: Liver cell adenoma, bile duct adenoma, haemangioma

342
Q

What is the most common malignant tumour of the liver?

A

Secondary tumour

343
Q

Name 4 primary malignant tumours of the liver?

A
  1. Hepatocellular carcinoma
  2. Hepatoblastoma
  3. Cholangiocarcinoma
  4. Haemangiosarcoma
344
Q
Which of the following is not associated with cholangiocarcinoma:
A. PSC
B. PBC
C. Worm infection 
D. Cirrhosis
A

B. PBC

All the other are associated with cholangiocarcinoma

345
Q

What is the typical triad of symptoms seen with budd chiari syndrome?

A

Abdominal pain, ascites and hepatomegaly

It is an occlusion of the hepatic veins that drain the liver

346
Q

Name three complications of Pelvic inflammatory disease

A
  1. Infertility
  2. Peritonitis
  3. Bacteraemia
347
Q

What strains of HPV are high risk for cervical cancer?

A

HPV 16 and 18

348
Q

What strains of HPV are low risk and often cause genital and oral warts?

A

HPV 6 and 11

349
Q

What are the two types of cervical cancer?

A

Adenocarcinoma and squamous cell carcinoma

350
Q

In cervical screening, what does CIN3 refer to:
A. Abnormal cells limited to the superficial third of the epithelium
B. Abnormal cells limited to 2 thirds of the epithelium
C. Abnormal cells limited to more than 2/3rd epithelium
D. Full thickness but no penetration of surrounding tissue

A

C. Abnormal cells limited to more than 2/3rd epithelium

A = CIN1
B = CIN2
C = CIN3
D = carcinoma in situ 
Invasive carcinoma = invasion through the BM
351
Q
When infected with HPV the infection can be:
A. Latent or active
B. Latent or productive
C. Mild or severe
D. Low risk or high risk
A

B. Latent or productive

Latent = HPV DNA continues to reside in basal cells but infectious virions are not produced
No cellular effects of HPV. Infection only identified by molecular methods

Productive = HPV NDA replication occurs independently of the host and produces infectious virions. It has characteristic cytological and histological features

352
Q

What is the most common uterine tumour?

A

Leiomyoma = smooth muscle tumour of the endometrium

353
Q

What are the classes of endometrial cancer?

A

Type 1 = endometriosis, mucinous, secretory adenocarcinoma. These are the most common and low grade tumours. They are often oestrogen dependent and seen in younger patients.

Type 2 = serous and clear cell carcinomas. Less common and high grade. They are often oestrogen independent and seen in older women

354
Q

How is endometrial cancer staged?

A
FIGO staging 
1 = confined to uterus
2 = spread to cervix
3 = spread to annexe, vagina, local lymph nodes
4 = distant spread
355
Q
Which of the following ovarian tumours is associated with endometriosis?
A. Serous 
B. Mucinous
C. Endometriod 
D. Clear cell
A

C and D

356
Q

List some types of ovarian cysts

A

Non neoplastic functional cysts = follicular, luteal, endometriotic cysts
Cysts from polycystic disease

357
Q

What are the classes of ovarian tumours?

A

Type 1 = low grade, serous, endometriod, mucinous, CCC

Type 2 = high grade, mostly serous

358
Q

What is the most common type of ovarian tumour?

A

Serous

359
Q
What type of tumour is a thecoma?
A. Ovarian tumour
B. Germ cell tumour
C. Sex cord stromal tumour
D. Cervical tumour
A

C. Sex cord stromal tumour

A thecoma is made up of bundles of spindle cells and lipids. It is benign.
Others include: fibroma, granulosa, sertoli leydig cell tumour

360
Q

What is a Krukenburg tumour?

A

Secondary ovarian tumour.

Bilateral metastases composed of mucin producing signet ring cells.

Most often gastric origin or breast

361
Q
A lady presents with nipple discharge which is sent off for cytology. It comes back with proteinaceous material and inflammatory cells only and no increased risk of malignancy. What is the diagnosis?
A. Acute mastitis
B. Phyllodes tumour
C. Duct ectasia
D. Fibrocystic disease
A

C. Duct ectasia

Inflammation and dilation of the large breast ducts

362
Q
Which of the following is not a benign breast disease?
A. Intraductal papilloma
B. Radial scar
C. Ductal carcinoma 
D. Phyllodes tumour
A

C. Ductal carcinoma

363
Q

A benign breast disease seen in women aged 40-60 and often presenting with bloody discharge?

A

Intraductal papilloma

364
Q

A benign sclerosing lesion

A

Radial scar

Central zone of scarring surrounded by a radiating zone of proliferating glandular tissue

365
Q
Which of the following has a risk of going on to develop into invasive breast carcinoma? 
A. Duct ectasia
B. Radial scar
C. Intraductal papilloma
D. Flat epithelial atypia
E. Acute mastitis
A

D. Flat epithelial atypia

Proliferative breast disease include:

  • Unusual epithelial hyperplasia
  • Flat epithelial atypia
  • In situ lobular neoplasia
366
Q

Which of the following proliferative breast disease has the highest risk of developing into invasive breast carcinoma?
A. Unusual epithelial hyperplasia
B. Flat epithelial atypia
C. In situ lobular neoplasia

A

C. In situ lobular neoplasia

Has a 7-12x increased risk

367
Q

What is the receptor status of basal like carcinoma?
A. ER/PR negative, Her 2 negative
B. ER/PR positive, Her2 negative
C. ER/PR negative, Her2 positive

A

A. ER/PR negative, Her 2 negative

Basal cell carcinoma is triple negative

B. Low grade
C. High grade

368
Q

What 3 histological features are used to grade invasive breast cancers?

A
  1. Tubule formation
  2. Nuclear pleomorphism
  3. Mitotic activity
369
Q

What is the age group invited to breast screening programme?

A

47-73 years = screened every 3 years

370
Q
If a core biopsy of a breast lump came back as B3, what would this mean?
A. Suspicious of malignancy 
B. Invasive malignancy 
C. Benign 
D. Normal 
E. Uncertain malignant potential
A

E. Uncertain malignant potential

B1 = normal 
B2 = benign 
B3 = lesion of uncertain malignant potential 
B4 = suspicious of malignancy 
B5 = malignant (B5a = DCIS; B5b = invasive)
371
Q

What are the 2 types of pancreatic carcinoma and which is most common?

A

Ductal and acinar carcinoma. Ductal is most common

372
Q

IgG4 related disease is also known as?

A

Autoimmune pancreatitis

373
Q

What mutation is seen in 95% of ductal carcinoma?

A

k-ras

374
Q

Define metaplasia and dysplasia?

A
Metaplasia = Reversible change of one differentiated cell type into another differentiated cell type. 
Dysplasia = changes showing some of the cytological and histological features of malignancy but no invasion through basement membrane.
375
Q

What is the staging system used for colorectal cancer and what are the stages?

A
Dukes Staging
A = confined to wall of bowel 
B = through wall of bowel 
C = lymph node metastasis 
D = distant metastasis
376
Q

What are the two types of cerebral oedema?

A
Vasogenic = disruption of BBB
Cytotoxic = secondary to cellular injury (hypoxia, ischaemia)
377
Q

Well defined malformative lesion composed of closely packed vessels with no parenchyma interposed between vascular spaces:
A. Arteriovenous malformation
B. Non traumatic intra parenchymal haemorrhage
C. Cavernous angioma
D. Hydrocephalus

A

C. Cavernous angioma

Can be anywhere in the CNS.
Presents with headaches, seizures, focal deficits, haemorrhage and is treated with surgery

378
Q

What is the most common primary CNS tumour?
A. Glial tumour
B. Medulloblastoma
C. Meningioma

A

A. Glial tumour

379
Q
Which of the following tumours can progress to a glioblastoma?
A. Oligodendroglioma
B. Astrocytoma
C. Meningioma 
D. Medulloblastoma
A

B. Astrocytoma